NLE Compilation 1

67
1 NURSING PRACTICE I SET A ________________________________________________________________________ NURSING PRACTICE I – Foundation of PROFESSIONAL Nursing Practice GENERAL INSTRUCTIONS: 1. This test booklet contains 100 test questions. 2. Read INSTRUCTIONS TO EXAMINEES printed on your answer sheet. 3. Shade only one (1) box for each question on your answer sheets. Two or more boxes shaded will invalidate your answer. 4. AVOID ERASURES. 5. This is PRC property. Unauthorized possession, reproduction, and/or sale of this test is punishable by law. Per RA 8981. ________________________________________________________________________ INSTRUCTIONS: 1. Detach one (1) answer sheet from the bottom of your Examinee ID/Answer Sheet Set . 2. Write the subject title “Nursing Practice I” on the box provided. 3. Shade Set Box “A” on your answer sheet if your test booklet is Set A; Set Box “B” if your test booklet is Set B. MULTIPLE CHOICE 1. Nurse Suzie is administering 12:00 PM medication in Ward 4. Two patients have to receive Lanoxin. What should Nurse Suzie do when one of the clients does NOT have a readable identification band? A. Ask the client if she is Mrs. Santos B. Ask the client his name C. Ask the room mate if the client is Mrs. Santos D. Compare the ID band with the bed tag 2. Lizette, a head nurse in a surgical unit, hears one of the staff nurses say that she does not touch any client assigned to her unless she performs nursing procedures or conducts physical assessment. To guide the staff nurse in the use of touch, which of the following would be BEST response of Lizette? A. “Use touch when the situation calls for it”. B. “Touch serves as a connection between the nurse and the patient”. C. “Use touch with discretion”. D. “Touch is used in physical assessment”. 3. You are asked to teach the client, Mr. Lapuz, who has right sided weakness the use of a cane. Which observation will indicate that Mr. Lapuz is using the cane correctly? A. The cane and one foot or both feet are on the floor at all times B. He advances the cane followed by the left leg C. Client keeps the cane on the right side along the weak leg D. Client leans to the left side which is stronger 4. George, a 43 year old executive is scheduled for cardiac bypass surgery. While being prepared for the surgery, he says to the nurse “I am not going to have the surgery. I may die because of the risk.” Which response by the nurse is most appropriate? A. “Without the surgery you will most likely die sooner.” B. “There are always risks involved with surgery.” C. “There is a client in the other room who had successful surgery and you can talk to him.” D. “This must be very frightening for you. Tel me how you feel about the surgery.”

Transcript of NLE Compilation 1

Page 1: NLE Compilation 1

1

NURSING PRACTICE I SET A ________________________________________________________________________ NURSING PRACTICE I – Foundation of PROFESSIONAL Nursing Practice GENERAL INSTRUCTIONS:

1. This test booklet contains 100 test questions. 2. Read INSTRUCTIONS TO EXAMINEES printed on your answer sheet. 3. Shade only one (1) box for each question on your answer sheets. Two or more boxes shaded

will invalidate your answer. 4. AVOID ERASURES. 5. This is PRC property. Unauthorized possession, reproduction, and/or sale of this test is

punishable by law. Per RA 8981. ________________________________________________________________________ INSTRUCTIONS:

1. Detach one (1) answer sheet from the bottom of your Examinee ID/Answer Sheet Set . 2. Write the subject title “Nursing Practice I” on the box provided. 3. Shade Set Box “A” on your answer sheet if your test booklet is Set A; Set Box “B” if your test

booklet is Set B. MULTIPLE CHOICE

1. Nurse Suzie is administering 12:00 PM medication in Ward 4. Two patients have to receive Lanoxin. What should Nurse Suzie do when one of the clients does NOT have a readable identification band?

A. Ask the client if she is Mrs. Santos B. Ask the client his name C. Ask the room mate if the client is Mrs. Santos D. Compare the ID band with the bed tag

2. Lizette, a head nurse in a surgical unit, hears one of the staff nurses say that she does not

touch any client assigned to her unless she performs nursing procedures or conducts physical assessment. To guide the staff nurse in the use of touch, which of the following would be BEST response of Lizette?

A. “Use touch when the situation calls for it”. B. “Touch serves as a connection between the nurse and the patient”. C. “Use touch with discretion”. D. “Touch is used in physical assessment”.

3. You are asked to teach the client, Mr. Lapuz, who has right sided weakness the use of a cane.

Which observation will indicate that Mr. Lapuz is using the cane correctly?

A. The cane and one foot or both feet are on the floor at all times B. He advances the cane followed by the left leg C. Client keeps the cane on the right side along the weak leg D. Client leans to the left side which is stronger

4. George, a 43 year old executive is scheduled for cardiac bypass surgery. While being

prepared for the surgery, he says to the nurse “I am not going to have the surgery. I may die because of the risk.” Which response by the nurse is most appropriate?

A. “Without the surgery you will most likely die sooner.” B. “There are always risks involved with surgery.” C. “There is a client in the other room who had successful surgery and you can talk to

him.” D. “This must be very frightening for you. Tel me how you feel about the

surgery.”

Page 2: NLE Compilation 1

2

5. A client is ordered to take Lasix, a diuretic, to be taken orally daily. Which of the following is an appropriate instruction by the nurse?

A. Report to the physician the effects of the medication on urination. B. Take the medicine early in the morning C. Take a full glass of water with the medicine D. Measure frequency of urination in 24 hours

6. Nurse Glenda gets a call from the neighbor who tells her that his 3 years old daughter has

been vomiting and has fever and asks for advice. Which of the following is the most appropriate action of the nurse?

A. Observe the child for an hour. If the child does not improve, refer to the physician in

the neighborhood. B. Recommend to bring the child immediately to the hospital C. Assess the child, recommend observation and administer acetaminophen. If

symptoms continue, bring to the hospital. D. Tell the neighbor to observe the child and give plenty of fluids. If the child does not

improve, bring the child to the hospital.

7. Wilfred, 30 years old male, was brought to the hospital due to injuries sustained from a vehicular accident. While being transported to the X-ray department, he straps accidentally broke and the client fell to the floor hitting to his head. In this situation, the nurse is:

A. not responsible because of the doctrine of respondent superior B. free from any negligence that caused harm to the patient C. liable along with the employer for the use of a defective equipment that

harms the client D. totally responsible for the negligence

8. While going on evening round, Nurse Edna saw Mrs. Pascual meditating and afterwards

started singing prayerful hymns. What is the BEST response of Edna?

A. Ignore the incidence B. Report the incidence to the head nurse C. Respect the client’s actions as this provides structure and support to the

client D. Call her attention so she can go to sleep

9. A client asks for advice on low cholesterol food. You advise the client to eat the following:

A. Chicken liver, cow liver, eggs B. Lean beef and pork, egg ewhite, fish C. Balut, salted eggs, duck and chicken egg D. Pork liempo, cow brain, lungs and kidney

10. The code of ethics for nurses has an interpretative statement that provides:

A. continuity of care for the improvement of the client B. guide for carrying out nursing responsibilities that provide quality care and

for the ethical obligation of the profession C. standards of care in carrying out nursing responsibilities D. identical care to all clients in any setting

11. Which of the following situations would possibly cause a nurse to be sued due to negligence?

A. Nurse gave a client wrong medication, and an hour later, client complained of dyspnea

B. While preparing a medication, the nurse notices that instead of 1 tablet, she put two tablets into the client’s medicine cup

C. As the nurse was about to administer medication, the client questioned why the medication is still given when in fact the physician discontinued it.

D. Nurse administered 2 tablets of analgesic instead of 1 tablet as prescribed. Patient noticed the error and complained.

Page 3: NLE Compilation 1

3

12. Your nurse supervisor asks you who among the following clients is most susceptible to getting

infection if admitted to the hospital?

A. Diabetic client type2 B. Client with chronic obstructive pulmonary disease (COPD) C. Client with second degree burns D. Client with psoriasis

13. Mr. Chris Martinez has been confined for three days. His wife helped take care of him and he

has observed her to be “too involved” in his care. He complained to the head nurse about this. Which of the following would be the BEST response of the nurse?

A. “Don’t worry. I will call the attention of your wife.” B. “Your wife is just trying to help because she is worried about you.” C. “What are your thoughts about your wife’s involvement in your care?” D. “Your wife can assist you well in your care and recovery.”

14. The nurse is in the hospital canteen and hears two staff nurses talking about the client

confined in Room 612. They mentioned his name and discussed details of his condition. Which of the following actions should the nurse take?

A. Approach the two nurses and tell them that their actions are inappropriate

especially in a public place B. Wait till the nurses finish the discussion and report the situation to the supervisor C. Say nothing to avoid embarrassing the staff nurses D. Remain quiet and ignore the discussion

15. The son of Mr. Rosario, a 76 year old man, reports to the nurse in the community health

center that his father has been getting out of bed at night and walks around the house in the early hours of the morning causing him to fall and injure himself. Which instruction would you give?

A. Apply restraints during night hours only B. Advise hospitalization to prevent future accidents C. Keep a radio or TV for company and to orient the client D. Have someone check on the client frequently at night

SITUATIONAL Situation 1 – Preparation and administration of medications is a nursing function that cannot be delegated. It is important that the nurse has a deep understanding of this responsibility that is meant to save patient’s lives. 16. You are to administer an intramuscular injection to Dulce, 1 ½ year old girl. The most

appropriate site to administer the drug is:

A. dorso gluteal region C. vastus lateralis B. ventral forearm D. gluteal region

17. An infant is ordered to receive 500ml of D5NSS for 24 hours. The intravenous drip is running

at 60 drops/minute. How many drops per minute should the flow rate be?

A. 60 drops per minute C. 30 drops per minute B. 21 drops per minute D. 15 drops per minute

18. Following surgery, Henry is to receive 20 mEq (milliequivalent) of potassium chloride to be

added to 1000 ml of D5W to run for 8 hours. The intravenous infusion set is calibrated at 20 drops per milliliter. How many drops per minute should the rate be to infuse 1 liter of D5W for 8 hours?

A. 42 drops C. 60 drops B. 20 drops D. 32 drops

Page 4: NLE Compilation 1

4

19. Mr. Lagro is to receive 1 liter of D5LR to run for 12 hours. The drop factor of the IV infusion set is 10 drops per minute. Approximately how many drop per minutes should the IV be regulated?

A. 13-14 drops C. 10-12 drops B. 17-18 drops D. 15-16 drops

20. The physician ordered Nembutal Na gr XX. The bottle contains 100mg/capsule. How many capsule will be administered to the client?

A. 1 capsule C. 2 capsule B. 1 ½ capsule D. ½ capsule

Situation 2 – The nurse supervisor is observing the staff nurses in her hospital to see how quality of care provided to clients can be improved.

21. The nurse supervisor is not satisfied with the bed bath that is provided by Nurse Arthur. To

improve the care provided to the patients in the unit by Nurse Arthur, the nurse supervisor should:

A. tell the nurse how to give bed baths correctly B. ask another staff nurse to do bed baths instead C. provide a manual to be read on giving bed baths D. bring the staff nurse to a client’s room and demonstrate

22. The staff nurse discusses with the novice nurse the type of wound dressing that is best to

use for a client. Together, they observe how well the dressings absorb the drainage. In what step of the decision making process are they?

A. Testing options C. Defining the problem B. Considering effects on results D. Making final decisions

23. To check if the nurses under her supervision use critical thinking, Mrs. David observes if the

nurses act responsibly when at work. Which of the following actions of the nurse demonstrates the attitude of responsibility?

A. Thinking of alternative methods of nursing care B. Sharing ideas regarding patient care C. Following standards of practice D. Planning other approaches for patient care

24. The nurse who makes clinical judgment can be depended upon to improve the quality of care

of clients. Nurse Julie uses such good clinical judgment when she gives priority care to this client:

A. Roman, a client who is ambulatory and for surgery tomorrow B. A post operative client, Rey, who has a blood pressure of 90/50 mmHg C. Mr. Abad, a client who needs instructions for home medications D. Fred, a client who received pain medications 5 minutes ago

25. A good nursing care plan is dependent on a correctly written nursing diagnosis. It defines a

client’s problem and its possible cause. The following is an example of a well written nursing diagnosis:

A. Acute pain related to altered skin integrity secondary to hysterectomy B. Electrolyte imbalance related to hypocalcemia C. Altered nutrition related to high fat intake secondary to obesity D. Knowledge deficit related to proctosigmoidoscopy

Situation 3– You are taking care of Mrs. Leyba, 66 years old, who is terminally ill with ovarian cancer stage IV. 26. When caring for a dying client, you will perform which of the following activities?

A. Encourage the client to reach optimal health B. Assist client perform activities of daily living

Page 5: NLE Compilation 1

5

C. Assist the client towards a peaceful death D. Motivate client to gain independence

27. The client prepares for her eventual death and discusses with the nurse and her family how

she would like her funeral to look like and what dress she will use. This client is in the stage of:

A. acceptance C. denial B. resolution D. bargaining

28. The nurse is to administer Demerol 50 mg IM to Mrs. Leyba. Demerol is available in a

mutidose vial labelled 100 mg/ml and Vistaril comes in an ampule labelled 50 mg/ml. You are to give the both medications in one injection. You will:

A. withdraw the medication from the vial first then from the ampule B. inject air into the vial, then into the ampule C. inject air into the ampule, aspirate the desired dose, then into the vial D. withdraw medication from the ampule then from the vial

29. When giving Demerol 50 mg from a multidose vial labelled 100 mg/ml and Vistaril 50 mg/ml from an ampule labelled 50 mg/ml, what is the total volume that you will inject to the client?

A. 2 ml C. 1.5 ml B. 1 ml D. 1.75 ml

30. Mrs. Leyba is emaciated and is at risk for developing which problem in skin integrity?

A. Blisters C. Pressure sores B. Reddening of the skin D. Pustules

Situation 4 – You are assigned to work in an orthopedic ward where clients are expected to have problems in mobility and immobility.

31. Ramil’s right leg is injured and Nurse Karen has to move him from the bed to w wheel chair.

Which of the following is the appropriate nursing action of Nurse Karen?

A. Put the client on the edge of the bed and place the wheelchair at her back B. Face the client and place the wheelchair on her left side C. Put the client on the edge of the bed and place the wheelchair on the other side of

the bed D. Put the client on the edge of the bed and place the wheelchair on the client’s

left side

32. Carlo has to be maintained on a dorsal recumbent position. Which of the following should be prevented?

A. adduction of the shoulder B. Lateral flexion of the sternocleidomastoid muscle C. Hyperextension of the knees D. Anterior flexion of the lumbar curvature

33. Joseph prefers to be in high fowler’s position most of the time. The nurse should prevent which of the following?

A. Posterior flexion of the lumbar curvature B. Internal rotation of the shoulder C. External rotation of the hip D. Adduction of the shoulder

34. Anthony asks to be assisted to move up the bed. Which of the following should Nurse Diana

do first?

A. Move the patient to the edge of the bed near the nurse B. Adjust the bed to flat position C. Lock the wheels of the bed D. Raise the bed rails opposite the nurse

Page 6: NLE Compilation 1

6

35. Which of the following supportive devices can be used most effectively by Nurse Arnold to

prevent external rotation of the right leg?

A. Sandbags C. Pillow B. Firm mattress D. High foot board

Situation 5 – As you begin to work in the hospital where you are on probation, you are tasked to take care of a few patients. The clients have varied needs and you are expected to provide care for them.

36. An ambulatory client. Mr. Zosimo, is being prepared for bed. Which of the following nursing actions promote safety for the client?

A. Turning off the lights to promote rest and sleep B. Instructing the client about the use of call system C. Raising the side rails D. Placing the bed in high position

37. Mikka, a 25 year old female client, is admitted with right lower quadrant abdominal pain. The physician diagnosed the client with acute appendicitis and an emergency appendectomy was performed. Twelve hours following surgery, the patient complained of pain. Which of the following is the most appropriate nursing diagnosis?

A. Impaired mobility related to pain secondary to an abdominal incision B. Impaired movements related to pain due to surgery C. Impaired mobility related to surgery D. Severe pain related to surgery

38. You are preparing a plan of care for a client who is experiencing pain related to incisional swelling following laminectomy. Which of the following should be included in the nursing care plan?

A. Encourage the client to log roll when turning B. Encourage the client to do self-care C. Instruct the client to do deep breathing exercises D. Ambulate the client in ward premises every twenty minutes

39. Mr. Lozano, 50 year old executive, is recovering from severe myocardial infarction. For the past 3 days, Mr. Lozano’s hygiene and grooming needs have been met by the nursing staff. Which of the following activities should be implemented to achieve the goal of independence for Mr. Lozano?

A. Involving family members in meeting client’s personal needs B. Meeting his needs till he is ready to perform self-care C. Preparing a day to day activity list to be followed by client D. Involving Mr. Lozano in his care

40. Mr. Ernest Lopez is terminally ill and he choose to be at home with his family. What nursing action are best initiated to prepare the family of Mr. Lopez?

A. Talk with the family members about the advantage of staying in the hospital for

proper care B. Provide support to the family members by teaching ways to care for their

loved one C. Convince the client to stay in the hospital for professional care D. Tell the client to be with his family

Situation 6 – Myrna, a researcher, proposes a study on the relationship between health values and the health promotion activities of staff nurses in a selected college of nursing.

Page 7: NLE Compilation 1

7

41. In both quantitative and qualitative research, the used of a frame of reference is required. Which of the following items serves as the purpose of a framework?

A. Incorporates theories into nursing’s body of knowledge B. Organizes the development of study and links the findings to nursing’s body

of knowledge C. Provides logical structure of the research findings D. Identifies concepts and relationships between concepts

42. Myrna need to review relevant literature and studies. The following processes are undertaken

in reviewing literature EXCEPT:

A. locating and identifying resources C. clarifying a research topic B. reading and recording notes D. using the library

43. The primary purpose for reviewing literature is to:

A. organize materials related to the problem of interest B. generate broad background and understanding of information related to the

research problem of interest C. select topics related to the problem of interest D. gather current knowledge of the problem of interest

44. In formulating the research hypotheses, researcher Myrna should state the research question as:

A. What is the response of the staff nurses to the health values? B. How is variable “health value” perceived in a population? C. Is there a significant relationship between health values and health

promotion activities of the staff nurses? D. How do health values affect health promotion activities of the staff nurses?

45. The proposed study shows the relationship between the variables. Which of the following is the independent variable?

A. Staff nurses in a selected college of nursing B. Health values C. Health promotion activities D. Relationship between health values and health promotion activities

Situation 7 – While working in a tertiary hospital, you are assigned to the medical ward.

46. Your client, Mr. Diaz, is concerned that he can not pay his hospital bills and professional fees.

You refer him to a:

A. Nurse supervisor C. bookkeeping department B. Social worker D. physician

47. Mr. Magno has lung cancer and is going through chemotherapy. He is referred by the oncology nurse to a self-help group of clients with cancer to:

A. receive emotional support C. provide financial assistance B. to be a part of a research study D. assist with chemotherapy

48. A diabetic hypertensive client, Mrs. Linao, needs a change in diet to improve her health status. She should be referred to a:

A. nutritionist C. physician B. dietitian D. medical pathologist

49. When collaborating with other health team members, the best description of Nurse Rita’s role

is: A. encourages the client’s involvement in his care B. shares and implements orders of the health team to ensure quality care

Page 8: NLE Compilation 1

8

C. she listens to the individual views of the team members D. helps client set goals of care and discharge

50. Nurse Rita is successful in collaborating with health team members about the care of Mr. Linao. This is because she has the following competencies:

A. Communication, trust, and decision making B. Conflict management, trust, negotiation C. Negotiation, decision making D. Mutual respect, negotiation and trust

Situation 8 – The practice of nursing goes with responsibilities and accountability whether you work in a hospital or in the community setting you main objective is to provide safe nursing to your clients?

51. To provide safe, quality nursing care to various clients in any setting, the most important tool of the nurse is:

A. critical thinking to decide appropriate nursing actions B. understanding of various nursing diagnoses C. observation skills for data collection D. possession of in scientific knowledge about client needs

52. You ensure the appropriateness and safety of your nursing interventions while caring for various client groups by:

A. creating plans of care for particular clientele B. identifying the correct nursing diagnoses for clients C. making a thorough assessment of client needs and problems D. using standards of nursing care as your criteria for evaluation

53. The effectiveness of your nursing care plan for your clients is determined by

A. the number of nursing procedures performed to comfort the client B. the amount of medications administered to the client as ordered C. the number of times the client calls the nurse D. the outcome of nursing interventions based on plan of care

54. You are assigned to Mrs. Amado, age 49, who was admitted for possible surgey. She complained of recurrent pain at the right upper quadrant of the abdomen 1-2 hours after ingestion of fatty food. She also had frequent bouts of dizziness, blood pressure of 170/100, hot flashes. Which of the above symptoms would be an objective cue?

A. Blood pressure measurement of 170/100 B. Complaint of hot flashes C. Report of pain after ingestion of fatty food D. Complaint of frequent bouts of dizziness

55. While talking with Mrs. Amado, it is most important for the nurse to:

A. schedule the laboratory exams ordered for her B. do an assessment of the client to determine priority needs C. tell the client that your shift ends after eight hours D. have the client sign an informed consent

Situation 9 – Oral care is an important part of hygienic practices and promoting client comfort.

56. An elderly client, 84 years old, is unconscious. Assessment of the mouth reveals excessive dryness and presence of sores. Which of the following is BEST to use for oral care?

A. lemon glycerine C. Mineral oil B. hydrogen peroxide D. Normal saline solution

Page 9: NLE Compilation 1

9

57. When performing oral care to an unconscious client, which of the following is a special

consideration to prevent aspiration of fluids into the lungs?

A. Put the client on a sidelying position with head of bed lowered B. Keep the client dry by placing towel under the chin C. Wash hands and observe appropriate infection control D. Clean mouth with oral swabs in a careful and an orderly progression

58. The advantages of oral care for a client include all of the following, EXCEPT:

A. decreases bacteria in the mouth and teeth B. reduces need to use commercial mouthwash which irritate the buccal

mucosa C. improves client’s appearance and self-confidence D. improves appetite and taste of food

59. A possible problem while providing oral care to unconscious clients is the risk of fluid

aspiration to lungs. This can be avoided by:

A. Cleaning teeth and mouth with cotton swabs soaked with mouthwash to avoid rinsing the buccal cavity

B. swabbing the inside of the cheeks and lips, tongue and gums with dry cotton swabs C. use fingers wrapped with wet cotton washcloth to rub inside the cheeks, tongue,

lips and ums D. suctioning as needed while cleaning the buccal cavity

60. Your client has difficulty of breathing and is mouth breathing most of the time. This causes dryness of the mouth with unpleasant odor. Oral hygiene is recommended for the client and in addition, you will keep the mouth moistened by using:

A. salt solution C. petroleum jelly B. water D. mentholated ointment

Situation 10 – Errors while providing nursing care to patients must be avoided and minimized at all time. Effective management of available resources enables the nurse to provide safe, quality patient care.

61. In the hospital where you work, increased incidence of medication error was identified as the number one problem in the unit. During the brainstorming session of the nursing service department, probable causes were identified. Which of the following is process related?

A. interruptions C. lack of knowledge B. use of unofficial abbreviations D. failure to identify client

62. Miscommunication of drug orders was identified as a probable cause of medication errors. Which of the following is safe medication practice related to this?

A. Maintain medication in its unit dose package until point of actual administration B. Note both generic and brand name of the medication in the Medication

Administration Method C. Only officially approved abbreviations maybe used in prescription orders D. Encourage clients to ask question about their medications.

63. The hospital has an ongoing quality assurance program. Which of the following indicates implementation of process standards?

A. The nurses check client’s identification band before giving medications B. The nurse reports adverse reaction to drugs C. Average waiting time for medication administration is measured D. The unit has well ventilated medication room

Page 10: NLE Compilation 1

10

64. Which of the following actions indicate that Nurse Jerome is performing outcome evaluation of quality care?

A. Interviews nurses for comments regarding staffing B. Measures waiting time for client’s per nurse’s call C. Checks equipment for its calibration schedule D. Determines whether nurses perform skin assessment every shift

65. An order for a client was given and the nurse in charge of the client reports that she has no experience of doing the procedure before. Which of the following is the most appropriate action of the nurse supervisor?

A. Assign another nurse to perform the procedure B. Ask the nurse to find way to learn the procedure C. Tell the nurse to read the procedure manual D. Do the procedure with the nurse

Situation 11 – Mr. Jose’s chart is the permanent legal recording of all information that relates to his health care management. As such, the entries in the chart must have accurate data.

66. Mr. Jose’s chart contains all information about his health care. The functions of records include

all except:

A. means of communication that health team members use to communicate their contributions to the client’s health care

B. the client’s record also shows a document of how much health care agencies will be reimbursed for their services

C. educational resource for student of nursing and medicine D. recording of actions in advance to save time

67. An advantage of automated or computerized client care system is:

A. The nursing diagnoses for a client’s data can be accurately determined B. Cost of confinement will be reduced C. Information concerning the client can b easily updated D. The number of people to take care of the client will be reduced

68. Information in the patient’s chart is inadmissible in court as evidence when:

A. The client’s family refuses to have it used B. The client objects to its use C. The handwriting is not legible D. It has too many abbreviations that are “unofficial”

69. Nursing audit aims to:

A. provide research data to hospital personnel B. study client’s illness and treatment regimen closely C. compare actual nursing done to established standards D. provide information to health-care providers

70. A telephone order is given for a client in your ward. What is your most appropriate action?

A. Copy the order on to the chart and sign the physician’s name as close to his original signature as possible

B. Repeat the order back to the physician, copy onto the order sheet and indicate that it is a telephone order

C. Write the order in the client’s chart and have the head nurse co-sign it D. Tell the physician that you can not take the order but you will call the nurse

supervisor

Page 11: NLE Compilation 1

11

Situation 12 – Nurse Roque, a newly hired nurse, is asked to take over an absent nurse in another unit. She will take care of clients with various conditions. 71. Which of the following client conditions should be Miss Roque’s priority in the pediatric unit?

A. The baby whose fantanelle is bulging and firm while asleep B. The infant who is brought in for upper respiratory tract infection whose

temperature is slightly elevated C. A baby who is wailing after being awakened by the banging door D. A baby boy whose circumcision has yellowish exudate

72. When suctioning the endotracheal tube, the nurse should:

A. Explain procedure to patient; insert catheter gently applying suction. Withdrawn using twisting motion

B. Insert catheter until resistance is met, then withdraw slightly, applying suction intermittently as catheter is withdrawn

C. Hyperoxygenate client insert catheter using back and forth motion D. Insert suction catheter four inches into the tube, suction 30 seconds using twirling

motion as catheter is withdrawn

73. Nurse Roque is giving instructions to Doris, the daughter of a comatose patient, to give a sponge bath. While Doris is doing spone bath, what action of Doris needs correction?

A. Answering the phone while wearing gloves used for sponge bath B. Rolling the patient like a log to do back rub C. Lining the rubber mat with bed sheet as incontinence pad for the patient D. Turning the patient on the left side with head slightly elevated

74. Dina sustained a fracture of the ulna and a cast will be applied. What nursing action before cast application is most important for Nurse Roque to do?

A. Use baby powder to reduce irritation under the cast B. Assess sensation of each arm C. Evaluate skin temperature in the area D. Check radial pulses bilaterally and compare

75. To obtain specimen for sputum culture and sensitivity, which of the following instruction is best?

A. Upon waking up, cough deeply and expectorate into container B. Cough after pursed lip breathing C. Save sputum for two days in covered container D. After respiratory treatment, expectorate into a container

Situation 13 – Infections are quite commonly the reasons for a client’s hospitalization. Appropriate interpretation of diagnostic tests and measures for infection control are helpful in the management of patient care.

76. Dorothy underwent diagnostic test and the result of the blood examination are back. On

reviewing the result the nurse notices which of the following as abnormal finding?

A. Neutrophils 60% B. White blood cells (WBC) 9000/mm C. Erythrocyte sedimentation rate (ESR) is 39 mm/hr D. Iron 75 mg/100 ml

77. Surgical sepsis is observed when:

A. inserting an intravenous catheter B. disposing of syringes and needles in puncture proof containers C. washing hands before changing wound dressing D. placing dirty soiled linen in moisture resistant bags

Page 12: NLE Compilation 1

12

78. A client with viral infection will most likely manifest which of the following during the illness stage of the infection?

A. Client was exposed to the infection 2 days ago but without any symptoms B. Oral temperature shows fever C. Acute symptoms are no longer visible D. Client “feels sick” but can do normal activities

79. Which of the following laboratory test result indicate presence of an infectious process?

A. Erythrocyte sedimentation rate (ESR) 12 mm/hr B. White blood cells (WBC) 18,000/mm3 C. Iron 90 g/100ml D. Neutrophils 67%

80. Among the clients you are assigned to take care of, who is the most susceptible to infection?

A. Diabetic client C. client with pulmonary emphysema B. Client with burns D. client with myocardial infarction

Situation 14 – You are a newly hired nurse in a tertiary hospital. You have finished your orientation program recently and you are beginning to assimilate the culture of the profession.

81. Using Benner’s stages of nursing expertise, you are a beginning nurse practitioner. You will rank yourself as a/an:

A. competent nurse C. proficient nurse B. novice nurse D. advanced beginner

82. Benner’s “Proficient” nurse level is different from the other levels in nursing expertise in the context of having:

A. the ability to organize and plan activities B. having attained an advanced level of education C. a holistic understanding and perception of the client D. intuitive and analytic ability in new situations

83. As you become socialized into the nursing “culture” you become a patient advocate. Advocacy is explained by the following EXCEPT:

A. respecting a person’s right to be autonomous B. demonstrating loyalty to the institution’s rights C. shared respect, trust and collaboration in meeting health needs D. protecting and supporting another person’s rights

84. Modern day nursing has led to the led development of the expanded role of the nurse as seen in the function of a:

A. Clinical nurse specialist C. community health nurse B. Critical care nurse D. staff nurse

85. You join a continuing education program to help you:

A. Earn credits for license renewal B. Get in touch with colleagues in nursing C. Enhance your basic knowledge D. Update your knowledge and skills related to field of interest

Page 13: NLE Compilation 1

13

Situation 15 – When creating your lesson plan for cerebrovascular disease or STROKE. It is important to include the risk factors of stroke.

86. The most important risk factor is:

A. Cigarette smoking C. binge drinking B. Hypertension D. heredity

87. Part of your lesson plan is to talk about etiology or cause of stroke. The types of stroke based on cause are the following EXCEPT:

A. Embolic stroke C. diabetic stroke B. Hemorrhagic stroke D. thrombotic stroke

88. Hemmorhagic stroke occurs suddenly usually when the person is active. All are causes of hemorrhage, EXCEPT:

A. phlebitis C. damage to blood vessel B. trauma D. aneurysm

89. The nurse emphasizes that intravenous drug abuse carries a high risk of stroke. Which drug is closely linked to this?

A. Amphetamines C. shabu B. Cocaine D. Demerol

90. A participant in the STROKE class asks what is a risk factor of stroke. Your best response is:

A. “More red blood cells thicken blood and make clots more possible.” B. “Increased RBC count is linked to high cholesterol.” C. “More red blood cell increases hemoglobin content.” D. “High RBC count increases blood pressure.”

Situation 16 – Accurate computation prior to drug administration is a basic skill all nurses must have.

91. Rudolf is diagnosed with amoebiasis and is to received Metronidazole (Flagyl) tablets 1.5 gm

daily in 3 divided doses for 7 consecutive days. Which of the following is the correct dose of the drug that the client will received per oral administration?

A. 1,000 mg tid C. 1,500 mg tid B. 500 mg tid D. 50 mg tid

92. Rhona, a 2 year old female was prescribed to receive 62.5 mg suspension three times a day. The available dose is 125 mg/ml. which of the following should Nurse Paolo prepare for each oral dose?

A. 0.5 ml C. 2.5 ml B. 1.5 ml D. 10 ml

93. The physician ordered Potassium Chloride (KCL) in D5W 1 liter to be infused in 24 hours for Mrs. Gomez. Since Potassium Chloride is a high risk drug, Nurse Robert used an intravenous pump. Which of the following should Nurse Robert do to safely administer this drug?

A. Check the pump setting every 2 hours B. Teach the client how the infusion pump operates C. Have another nurse check the infusion pump setting D. Set the alarm of the pump loud enough to be heard

94. Baby Liza, 3 months old, with a congenital heart deformity, has an order from her physician: “give 3.00 cc of Lanoxin today for 1 dose only”. Which of the following is the most appropriate action by the nurse?

A. Clarify order with the attending physician B. Discuss the order with the pediatric heart specialist in the unit

Page 14: NLE Compilation 1

14

C. Administer Lanoxin intravenously as it is the usual route of administration D. Refer to the medication administration record for previous administration of Lanoxin

95. When Nurse Norma was about to administer the medications of client Lennie, the relative of Lennie told the nurse that they buy her medicines and showed the container of medications of the client. Which of the following is the most appropriate action by the nurse?

A. Hold the nurse administration of the client’s medication and refer to the head nurse B. Put aside the medications she prepared and instead administer the client’s

medications C. Tell the client that she will inform the physician about this D. Bring the medications of the client to the nurse’s station and prepare accordingly

Situation 17 – You are taking care of Mrs. Santillan a 48 year old woman who is unconscious after a cerebrovascular accident. You are aware that there are many physical complications due to immobility.

96. You should be alert for the following complications she may experience EXCEPT:

A. Impaired mobility C. hypostatic pneumonia B. Contractures and muscle atrophy D. pressure sores

97. Proper positioning of an immobilized unconsciousness client is important for the following reasons EXCEPT:

A. Maintain skin integrity B. Promotes optimal lung expansion C. Prevent injuries and deformities of the musculo-skeletal system D. Facilitates rest and sleep

98. When positioning your client, you should observe good body mechanics for yourself and the client. This means that the nurse:

A. Uses back muscles B. Assumes correct body alignment and efficient use of muscles to avoid injury C. Observes rhythmic movements when moving about D. Uses large muscles only

99. You are going to move Mrs. Santillan, a 150 lbs unconscious woman. Some principls to use when moving the client include the following EXCEPT:

A. prepare to move client by taking deep breath and tightening abdominal and gluteal

muscles B. maintain wide base of support with feet and with knees flexed C. push and pull using arms and legs instead of lifting D. move close to the object to be moved leaning or bending at the waist

100. After moving Mrs. Santillan to the desired position, which action will you avoid?

A. Avoid friction between bony prominences B. Place pillows to position client’s extremeties C. Apply restraints D. Raise bed rails

SUBMIT THIS TEST BOOKLET TOGETHER WITH THE ANSWER SHEET TO YOUR WATCHERS. BRINGING THE TEST BOOKLET OUT OF THE ROOM WILL BE A GROUND FOR CANCELLATION OF YOUR EXAMINATION.

***END***

Page 15: NLE Compilation 1

15

NURSING PRACTICE II SET A _________________________________________________________________ NURSING PRACTICE II – Foundation of PROFESSIONAL Nursing Practice GENERAL INSTRUCTIONS:

6. This test booklet contains 100 test questions. 7. Read INSTRUCTIONS TO EXAMINEES printed on your answer sheet. 8. Shade only one (1) box for each question on your answer sheets. Two or more boxes shaded

will invalidate your answer. 9. AVOID ERASURES. 10. This is PRC property. Unauthorized possession, reproduction, and/or sale of this test is

punishable by law. Per RA 8981. _________________________________________________________________ INSTRUCTIONS:

4. Detach one (1) answer sheet from the bottom of your Examinee ID/Answer Sheet Set . 5. Write the subject title “Nursing Practice II” on the box provided. 6. Shade Set Box “A” on your answer sheet if your test booklet is Set A; Set Box “B” if your test

booklet is Set B. MULTIPLE CHOICE

1. Registered nurses can be identified as a:

A. Organization C. group B. Culture D. subculture

2. Among children candidates for organ transplant, when all selected children have appropriate tissue matches for the same donated organ, the basis for the decision as to which child gets the organ is given to the child who:

A. will receive the most benefit from the new organ B. is most likely to die without the transplant C. is selected by the lottery system for available organs D. is at the top of the list and has waited the longest time

3. The nurse uses what equipment to check for fluid between the parietal and visceral layers of the tunica vaginalis, the outermost covering of the testes?

A. 30 cc syringe C. Fluid meter B. transilluminator D. Manometer

4. Which of the following examples best defines the term role reversal?

A. a lazy person becomes very productive in the family B. the good child takes on a bad child role C. a person who has been a good provider quits his or her job D. the child assumes a caregiver role toward the caregiver

5. When a nurse breaches the duty of confidentiality, he or she can be disciplined by both the employer and the Board of Nursing. In addition to this discipline, he or she can:

A. be held responsible for any damages that result B. be fined by the federal government C. be sentenced for up to 1 year in jail D. immediately lose his or her nursing license

6. A strategy for a change that focuses on teaching workers new technology is:

Page 16: NLE Compilation 1

16

A. normative-reactive C. providing information B. training D. power coercive

7. The nurse knows that the occurrence of shoulder dystocia during labor is:

A. polyhydramnios C. preterm birth B. maternal age D. macrosomia

8. The nurse instructs the mother that when overstimulated the infant will:

A. show increased alertness and eye contact B. respond with coordinated, synchronous body movement C. look away to reduce the intensity of the interaction D. drift off to deep sleep to shut out the interaction

9. Some strategies to maintain professional health are listed below. Which is NOT necessarily correct?

A. Networking with others in the health care field B. Join a professional organization C. Goal setting D. Read fiction and non-fiction materials

10. The nurse is assessing an 8 month-old infant for head lag, pulling the infant by the hands from a supine to a sitting position. The head does not stay in line with the body when being pulled forward. Which of the following statements best represents the significance of this finding?

A. Head lag should not be tested until the child is over 1 year of age. B. Significant head lag after the age of 6 months may indicate brain injury and

needs further investigations. C. The nurse has not conducted the test correctly and must do it again using proper

technique. D. This is a normal finding, as the infant’s head will not stay in line until after 8 months of

age.

11. Which statement is correct regarding the use of the cervical cap?

A. It may affect Pap smear results. B. It does not need to be fitted by the physician. C. It does not require the use of spermicide. D. It must be removed within 24 hours.

12. The major components of the communication process are:

A. verbal, written and nonverbal B. speaker, listener and reply C. facial expression, tone of voice and gestures D. message, sender, channel, receiver and feedback

13. The extent of burns in children are normally assessed and expressed in terms of:

A. the amount of body surface that is unburned B. percentages of total body surface area (TBSA) C. how deep the deepest burns are D. the severity of the burns on a 1 to 5 burn scale.

14. The school nurse notices a child who is wearing old, dirty, poor-fitting clothes; is always hungry; has no lunch money; and is always tired. When the nurse asks the boy his tiredness, he talks of playing outside until midnight. The nurse will suspect that this child is:

A. being raised by a parent of low intelligence quotient (IQ) B. an orphan C. a victim of child neglect D. the victim of poverty

Page 17: NLE Compilation 1

17

15. Which of the following indicates the type(s) of acute renal failure?

A. Four types: hemorrhagic with and without clotting, and nonhemorrhagic with and without clottings

B. One type: acute C. Three types: prerenal, intrarenal and postrenal D. Two types: acute and subacute

16. A means of facilitating professional staff development is by building upon skills, abilities, and of experience of each practitioner is called:

A. the novice to expert model C. career enhancement B. situational leadership model D. clinical ladder

17. Which of the following questions by the nurse would be best fit the philosophy of the nursing mutual participation model of care (NMPMC)?

A. “Have you brushed your child’s teeth today?” B. “How does your child look to you today?” C. “Where have you been all morning?” D. “Do you think your child’s color is worse”?

18. There are numerous definitions of the word “health”. Which definition below is from Florence Nightingale?

A. A state or a process of being and becoming an integrated and whole person B. The state of being free from illness or injury C. Being well and using every power the individual possesses to the fullest extent D. A state of complete physical, social, and mental well-being and not merely the absence

of disease or infirmity

19. Informal communication takes place when individuals talk and is best described by saying the participants:

A. are involved in a preexisting informal relationship B. talk with slang words C. have no particular agenda or protocol D. are relaxed

20. Tertiary care by the home health nurse is directed toward children with:

A. problems in mobility C. minor problems B. short-term needs D. clinically apparent disease

21. The endometrium thickens during which phase of the menstrual cycle?

A. Secretory phase C. Proliferative phase B. Menstrual phase D. Ischemic phase

22. A measurement tool to articulate the nursing workload for a specific patient or groups of patients over a specific period of time is called:

A. staffing pattern C. benchmarking B. skill mix D. patient classification

23. The mother of a 9 month-old infant is concerned that the head circumference of her baby is greater than the chest circumference. The BEST response by the nurse is:

A. “These circumference normally are the same, but in some babies this just differs.” B. “Perhaps your baby was small for gestational age or premature.” C. “This is normal until the age of 1 year, when the chest will be greater.”

Page 18: NLE Compilation 1

18

D. “Let me ask you a few questions, and perhaps we can figure out the cause of this difference.”

24. Which of the following approaches would work best when the nurse is communicating with an

infant?

A. Use an adult voice just as you would for anyone. B. Communicate through the caregivers. C. Allow the child time to warm up to the nurse. D. Respond only after the child cries for a while.

25. Evidence-based care started in medicine as a way to:

A. promote technological advances in medicine B. incorporate collaboration within all health care disciplines C. integrate individual experience with clinical research D. teach medical students the art and science of medicine

26. The nurse assessing newborn babies and infants during their hospital stay after birth will

notice which of the following symptoms as a primary manifestation of Hirschsprung’s disease?

A. A fine rash over the trunk B. Failure to pass meconium during the first 24 to 48 hours after birth C. The skin turns yellow and then brown over the first 48 hours of life D. High-grade fever

27. A client is 7 months pregnant and has just been diagnosed as having a partial placenta previa. She is stable and has minimal spotting and is being sent home. Which of these instructions to the client may indicate a need for further teaching?

A. Maintain bed rest with bathroom privileges B. Avoid intercourse for three days. C. Call if contractions occur. D. Stay on left side as much as possible when lying down.

28. Which of the following groups of people in the world disproportionately represents the homeless population?

A. Hispanics C. African Americans B. Asians D. Caucasians

29. The nurse notes that the infant is wearing a plastic-coated diaper. If a topical medication were to be prescribed and it were to go on the stomachs or buttocks, the nurse would teach the caregivers to:

A. avoid covering the area of the topical medication with the diaper B. avoid the use of clothing on top of the diaper C. put the diaper on as usual D. apply an icepack for 5 minutes to the outside of the diaper

30. The nurse assessing a child or adolescent with a diagnosis of dysrhytmic disorder would find which of the following symptoms?

A. Labile mood and hyperactive thyroid with an increase in circulating thyroid hormones

and associated symptoms B. Severe shaking of the hands when trying to hold a glass of water or other object C. A depression that is deeper, more acute, and more likely to lead to suicide than major

depressive disorder D. A depressed or irritable mood for most of the day, on most days, for 2 or more

years and low energy or fatigue.

Page 19: NLE Compilation 1

19

31. You were the nurse assigned to work with a child who has had whole brain radiation. You have assessed the child to be sleeping up to 20 hours a day and is having some nausea, malaise, fever and dysphasia. Based on this assessment, you are to work with the patients in which of the following areas.

A. Accepting a reoccurrence of the tumor B. Dealing with the side effects of radiation therapy C. Caring for the dying child D. Accepting the imminent death of their child

32. The nurse is planning interventions for a child who has inflammatory bowel disease (IBD) with a nursing diagnosis of “Nutrition: Less than body requirements.” Which of the following interventions will be most helpful in resolving this nursing problem?

A. Two large meals a day instead of several minimeals and snacks B. Special IBD diet (diet that has been proven effective for treating IBD) C. Salt-free diet high in potassium, vitamins and minerals D. Diet as tolerated with lactose hydrolyzed milk instead of milk products, and

omission of highly seasoned foods, and reduction of fiber

33. Emotional intelligence consists of a number of competencies. Some of these are listed below. Which is NOT a characteristic of emotional intelligence?

A. Self-esteem C. Empathy B. Self-awareness D. Self-regulation

34. Data collection for driving and restraining forces, including costs, desirability and feasibility, is a:

A. people issue C. political issue B. structural issue D. technology issue

35. One of four factors describing the experience of sexually abused children and the effect it has

on their growth and development is stigmatization that occurs when:

A. a child blames him or herself for the sexual abuse and begins to withdraw and isolate B. newspapers and the media don’t keep sexual abuse private and accidentally or on

purpose reveal the name of the victim C. the child has been blamed by the abuser for his or her sexual behaviors, saying that

the child asked to be touched or did not make the abuser to stop D. the child’s agony is shared by other members of the family or friends when the

sexual abuse becomes public knowledge

36. The painful phenomenon known as “back labor” occurs in a client whose fetus in what position?

A. Brow position C. Breech position B. Right Occipito-Anterior Position D. Left Occipito-Posterior

Position 37. FOCUS methodology stands for:

A. Focus, Organize, Clarify, Understand and Solution B. Focus, Opportunity, Continuous, Utilize, Substantiate C. Focus, Organize, Clarify, Understand, Substantiate D. Focus, Opportunity, Continuous (process), Understand, Solution

38. While community health nurses focus on the individual or the family, which of the following do they also have as their final objective?

A. The well-being of the chronically ill B. The financial well-being of the family C. The well-being of the extended family

Page 20: NLE Compilation 1

20

D. The well-being of the community

39. Which of the following is the best example of the ethical principle of fidelity?

A. Doing whatever the client or the client’s physician asks of you. B. Keeping a promise to return to the client’s room at a given time C. Being a good friend to the client by sharing secrets D. Saving the client time and money by not wasting supplies

40. Which of the following factors is most important in determining the success of relationships

used in delivering nursing care?

A. Type of illness of the client B. Transference and counter transference C. Effective communication D. Personality of the participants

41. Which of the following statements best describes the term glove type burn?

A. The parent was wearing heavy gloves or stockings on his or her hands while immersing the child in hot scalding water

B. The parents have dipped the child into hot liquid while he or she was asleep C. The child was wearing a glove when immersed in hot liquid D. The burn has the look of a glove immersed in hot scalding water.

42. The school nurse keeps a list of enrolled students who have medical or religious objections to immunizations and those who are likely to have decreased immunity. The nurse likely keeps this list to:

A. provide statistics for the Department of Health B. reassure the family that the nurse will respect the family’s wishes at all times C. meet national government requirements D. facilitate exclusion in case of an outbreak of a highly communicable disease in

the school.

43. Preschoolers are able to see things from which of the following perspectives?

A. Their peers B. Their own and their caregivers’ C. Their own and their mother’s D. Only their own

44. In conflict management, the win-win approach occurs when:

A. there are two conflicts and the parties agree to each one B. each party gives in on 50% of the disagreements making up the conflict C. both parties involved are committed to solving the conflict D. the conflict is settled out of court so the legal system and the parties win

45. According to the social-interactional perspective of child abuse and neglect, four factors place

the family members at risk for abuse. these risk factors are the family members at risk for abuse. These risk factors are the family itself, the caregiver, the child, and

A. the presence of a family crisis C. the national emphasis on sex B. genetics D. chronic poverty

46. When a person is discussing the strong influences that childrearing methods have on the development of the child, this person is most probably coming from which of the following viewpoints or theories?

A. Naturalistic C. Neoclassic B. Nature D. Nurture

Page 21: NLE Compilation 1

21

47. In working with the caregivers of a client with an acute or chronic illness, the nurse would:

A. Teach care daily and let the caregivers do a return demonstration just before discharge

B. Difficulty swallowing, diminished or absent gag reflex, and respiratory distress. C. Difficulty sleeping, hypervigilant, and an arching of the back D. Paradoxical irritability, diarrhea, and vomiting

48. Which of the following signs and symptoms would you most likely find when assessing and infant with Arnold-Chiari malformation?

A. Weakness of the leg muscles, loss of sensation in the legs, and restlessness B. Difficulty swallowing, diminished or absent gag reflex, and respiratory distress C. Difficulty sleeping, hypervigilant, and an arching of the back D. Paradoxical irritability, diarrhea, and vomiting.

49. A parent calls you and frantically reports that her child has gotten into her famous ferrous sulfate pills and ingested a number of these pills. Her child is now vomiting, has bloody diarrhea, and is complaining of abdominal pain. You will tell the mother to:

A. call emergency medical services (EMS) and get the child to the emergency room B. relax because these symptoms will pass and the child will be fine C. administer syrup of ipecac D. call the poison control center

50. A client says she heard from a friend that you stop having periods once you are on the “pill”.

The most appropriate response would be:

A. “The pill prevents the uterus from making such endometrial lining, that is why periods may often be scant or skipped occasionally.” B. “If your friend has missed her period, she should stop taking the pills and get a

pregnancy test as soon as possible.” C. “The pill should cause a normal menstrual period every month. It sounds like

your friend has not been taking the pills properly.” D. “Missed period can be very dangerous and may lead to the formation of precancerous

cells.”

51. You are the nurse assigned to work with a child with acute glomerulonephritis. By following the prescribed treatment regimen, the child experiences a remission. You are now checking to make sure the child does not have a relapse. Which finding would most lead you to the conclusion that a relapse is happening?

A. Elevated temperature, cough, sore throat, changing complete blood count (CBC) with

diiferential B. A urine dipstick measurement of 2+ proteinuria or more for 3 days, or the child

found to have 3-4+ proteinutria plus edema. C. The urine dipstick showing glucose in the urine for 3 days, extreme thirst, increase in

urine output, and a moon face. D. A temperature of 37.8 degrees (100 degrees F), flank pain, burning frequency, urgency

on voiding, and cloudy urine. 52. The nurse is working with an adolescent who complains of being lonely and having a lack of

fulfillment in her life. This adolescent shies away from intimate relationships at times yet at other times she appears promiscuous. The nurse will likely work with this adolescent in which of the following areas?

A. Isolation C. Lack of fulfillment B. Loneliness D. Identity

53. The use of interpersonal decision making, psychomotor skills, and application of knowledge expected in the role of a licensed health care professional in the context of public health welfare and safety is an example of:

A. delegation C. responsibility

Page 22: NLE Compilation 1

22

B. supervision D. competence

54. A child suffers a head injury in a tumbling accident in gym class. The nurse’s best course of action is to:

A. get the child up walking and make sure he or she stays awake B. leave the child and go get help C. leave the child in the care of an older child and go get help D. stay with the child, keep assessing, and have someone call the caregivers

55. The American Academy of Pediatrics suggests that caregivers do which of the following things in regard to physical activities for preschoolers?

A. Push the child to practice sports activities while they are more flexible B. Encourage a variety of physical activities in a noncompetitive environment C. Have the child engage in competitive sports to see where they excel D. Keep physical activities to a minimum until the child is in grade school.

56. Which of the following arrangements is generally considered to be best for the parents of hospitalized infant or young child?

A. Rooming-in B. Separate caregiver sleeping room on the unit C. Day visits and sleeping at home D. Staying at a nearby hotel or motel

57. When one person allows the conflict to be resolved at his or her own expense, this is referred to in conflict management as:

A. losing C. the win-lose approach B. winning while losing D. the lose-win approach

58. Which of the following statements best describes acquaintance rape?

A. Sexual intercourse when one person engaging in the activity is unsure about wanting to do so

B. When two people don’t love each other and engage in sexual activities C. When someone on a date tricks the other person into having sexual intercourse D. Sexual intercourse committed with force or the threat of force without a person’s

consent.

59. The school nurse is teaching a health education and hygiene course to a group of high school males, which includes a number of young men who are on competitive sports teams. Which of the following health practices would the nurse most stress in preventing the transmission of human immunodeficiency virus (HIV) virus in case any team member has HIV or acquired immunodeficiency syndrome (AIDS)?

A. No sharing of underarm deodorant or shower soap B. No sharing of razors or toothbrushes C. Making certain towels have been washed in boiling water D. Avoiding physical contact such as sports hugs or swats

60. At 17 weeks’ gestation, a type 1 diabetic undergoes an ultrasound examination. What

information about the fetus at this time in pregnancy would be the results of this examination provide?

A. Placental maturity C. Gestational age B. Estimated fetal weight D. Fetal lung maturity

61. Which of the following best describes a difference in communicating with school age children versus toddlers?

A. Toddlers require more empathy and more touching and holding

Page 23: NLE Compilation 1

23

B. For toddlers, preparation for procedures is just before the procedure and much earlier for school-aged children.

C. Caregivers need less information when care involves a school aged child D. The number of words is more when communicating with a toddler than it is with

a school aged children

62. Genetic testing should be performed on a child only if

A. the parents both want it performed B. it is in the best interests of the child C. it i9s necessary for the child to survive D. no one objects

63. Infant head control is judged by the: A. ability to hold the head without support B. presence or absence of head lag C. rigidity of the neck and head D. amount of neck wrinkling

64. Which of the following roles BEST exemplifies the expanded role of the nurse?

A. Circulating nurse in surgery B. Medication nurse C. Obstetrical nurse D. Pediatric nurse practitioner

65. The tone and pitch of the voice, volume, infection, speed, grunts and other vocalizations are referred to by which of the following terms?

A. Paraverbal clues C. Third element B. Ancillary speech D. Enhancements

66. The plan-do-study-act cycle begins with:

A. four stages C. three questions B. five agendas D. two concepts

67. During your shift, you noted one of your pregnant clients considered as “waiting case” manifest morning sickness and which later progressed. Which assessment finding may indicate possible developing complication?

A. Maternal pulse 90 C. FHT 155 B. Trace glucose in the urine D. 1+ ketones in the urine

68. The nurse is working with a child who is going to have a bone marrow aspiration. The physician orders TAC (tetracaine, adrenaline and cocaine). Which of the following is the route of administration?

A. application to the skin, covered with a dressing prior to the procedure B. subcutaneous C. IV using a very slow drip over approximately 4 hours prior to procedure D. Nasal inhalation

69. According to DeRosa and Kochura’s (2006) article entitled “Implement Culturally Competent Health Care in your workplace,” cultures have different patterns of verbal and nonverbal communication. Which difference does NOT necessarily belong?

A. Personal behavior C. Subject matter B. Eye contact D. Conversational style

70. The nurse instructs the caregivers of a newborn to notch the diapers or fold them in such a way as to expose the cord. The major purpose of exposing the cord is to:

A. remind caregivers to do cord care B. keep the diaper from rubbing the cord

Page 24: NLE Compilation 1

24

C. provide air circulation for the cord D. allow visualizations at all times

71. The level of health of an individual, family, group, population, or community is called:

A. health assets C. quality of life B. health status D. health needs

72. The nurse is teaching a group of expectant mothers about the prevention of diaper dermatitis. The nurse explains that one of the preventive measures is the use of:

A. absorbent disposable diapers C. plastic panties over diapers B. baby powder or cornstarch D. cloth diaper

73. The Code of Nurses

A. delineates all obligations and responsibilities of the nurse B. is a binding oath, which tells nurses how to make ethical decisions C. assists the nurse in formulating a personal belief system D. supports the concept of respect for all persons

74. The obligation to correctly perform one’s assigned duties is:

A. Delegation C. responsibility B. Assignment D. accountability

75. During a routine postpartum assessment following a normal vaginal delivery, the nurse notes the fundus to be slightly boggy. Which action should the nurse take to decrease the risk of uterine inversion during uterine massage?

A. Massage only when cramping begins. B. Place one hand on the abdomen above the symphysis pubis. C. Ask the client to ambulate to the bathroom to empty her bladder. D. Position the client in a slight Trendelenburg position.

SITUATIONAL Situation 1 – Nurse Lisa manages her own Reproductive and Children’s Nursing Clinic in Sorsogon and necessarily she attends to health conditions of mothers and children. The following conditions pertain to the GROWING FETUS.

76. Obstetrical client Marichu asks how much longer Nurse Lisa will refer to the baby inside her as an embryo. What would be your best explanation?

A. Her baby will be a fetus as soon as the placenta forms B. From the time of implantation until 5 to 8 weeks, the baby is an embryo C. After the 20th week of pregnancy, the baby is called a zygote D. This term is used during the time before fertilization

77. Marichu is worried that her baby will be born with a congenital heart disease. What assessment of a fetus at birth is important to help detect congenital heart defect?

A. Determining that the color of the umbilical cord if not green. B. Assessing whether the umbilical cord has two arteries and one vein. C. Assessing whether the Wharton’s jelly of the cord has a pH higher than 7.2. D. Measuring the length of the cord to be certain that it is longer than 3 feet.

78. Additionally, Nurse Lisa would gather more information about Marichu’s worry about what may threaten the health of her baby. What would Nurse Lisa hope to find?

A. Has Marichu been overly anxious about something. B. Has MArichu suffered from any communicable/contagious disease at the time of her

early stage of pregnancy.

Page 25: NLE Compilation 1

25

C. Has MArichu engaged in sexual activity during the fetal development state of her child. D. Has MArichu engaged in any detrimental activities during the fetal development

stage e.g. smoking, drinking, taking drugs, a bad fall, or attempts to terminate pregnancy.

79. Marichu is scheduled to have an ultrasound examination. What instruction would you give her

before her examination?

A. You can have medicine for pain for any contractions caused by the test. B. Drink at least 3 glasses of fluid before the procedure. C. The intravenous fluid infused to dilate your uterus does not hurt the fetus. D. Void immediately before the procedure to reduce your bladder size.

80. Marichu is scheduled to have an amniocentesis to test for fetal maturity. What instruction would you give her before this procedure?

A. The X-ray used to reveal your fetus’ position has no long term effects. B. The intravenous fluid infused to dilate your uterus does not hurt the fetus. C. No more amniotic fluid forms afterward, which is why only a small amount is removed. D. Void immediately before the procedure to reduce your bladder size.

Situation 2 – Health instructions are essentially given to pregnant mothers.

81. A public health nurse would instruct a pregnant woman to notify the physician immediately if

which of the following symptoms occur during pregnancy?

A. Presence of dark color in the neck B. Increased vaginal discharge C. Swelling of the face D. Breast tenderness

82. A woman who is 9 weeks pregnant comes to the Health Center with moderate bright red vaginal bleeding. On physical examination, the physician finds the client’s cervix 2 cm dilated. Which term best describes the client’s condition?

A. Missed abortion C. Inevitable abortion B. Incomplete abortion D. Threatened abortion

83. In a big government hospital, Nurse Pura is taking care of a woman with a diagnosis of abruption placenta. What complication of this condition is of most concern to Nurse Pura?

A. Urinary tract infection B. Pulmonary embolism C. Hypocalcemia D. Disseminated intravascular coagulation

84. Which of the following findings on a newly delivered woman’s chart would indicate she is at risk for developing postpartum hemorrhage?

A. Post-term delivery C. Grand multiparity B. Epidural anesthesia D. Premature rupture of membrane

85. Mrs. Hacienda Gracia, 35 year old postpartum client is at risk of thrombophlebitis. Which of the following nursing interventions decreases her chance of developing postpartum thrombophlebitis?

A. breastfeeding the newborn B. early ambulation C. administration of anticoagulant postpartum D. immobilization and elevation of the lower extremities

Page 26: NLE Compilation 1

26

Situation 3 – With the increasing documented cases of CANCER the best alternative to treatment still remains to be PREVENTION. The following conditions apply.

86. Which among the following is the primary focus of prevention of cancer?

A. Elimination of conditions causing cancer B. Diagnosis and treatment C. Treatment at early stage D. Early detection

87. In the prevention and control of cancer, which of the following activities is the most important function of the community health nurse?

A. Conduct community assemblies. B. Referral to cancer specialist those clients with symptoms of cancer. C. Use the nine warning signs of cancer as parameters in our process of detection,

control and treatment modalities. D. Teach woman about proper/correct nutrition.

88. Who among the following are recipients of the secondary level of care for cancer cases? A. Those under early case detection B. Those under post case treatment C. Those scheduled for surgery D. Those undergoing treatment

89. Who among the following are recipients of the tertiary level of care for cancer cases?

A. Those under early treatment C. Those under early detection B. Those under supportive care D. Those scheduled for surgery

90. In Community Health Nursing, despite the availability and use of many equipment and devices to facilitate the job of the community health nurse, the best tool any nurse should be wel be prepared to apply is a scientific approach. This approach ensures quality of care even at the community setting. This is nursing parlance is nothing less than the:

A. nursing diagnosis C. nursing research B. nursing protocol D. nursing process

Situation 4 – Dengue hemmorhagic fever is a common health concern in Philippine society. It does not only pose a threat to health but more so to the lives of both young and old, and well as rich and marginalized sectors of the society.

91. An important role of the community health nurse in the prevention and control of Dengue H-fever includes:

A. advising the elimination of vectors by keeping water containers covered B. conducting strong health education drives/campaign directed towards proper garbage

disposal C. explaining to the individuals, families, groups and community the nature of the

disease and its causation D. practicing residual spraying with insecticides

92. Community health nurses should be alert in observing a Dengue suspect. The following is NOT an indicator for hospitalization of H-fever suspects?

A. Marked anorexia, abdominal pain and vomiting B. Increasing hematocrit count C. Fever for more than 2 days D. Persistent headache

93. The community health nurses’ primary concern in the immediate control of hemorrhage among

patients with dengue is: A. advising low fiber and non-fat diet B. providing warmth through light weight covers

Page 27: NLE Compilation 1

27

C. observing closely the patient for vital signs leading to shock D. keeping the patient at rest

94. Which of these signs may NOT be REGARDED as a truly positive signs indicative of Dengue H-fever?

A. Prolonged bleeding time B. Appearance of at least 5 petechiae C. Steadily increasing hematocrit count D. Fall in the platelet count

95. Which of the following is the most important treatment of patients with Dengue H-Fever?

A. Give aspirin for fever B. Replacement of body fluids C. Avoid unnecessary movement of patient D. Ice cap over the abdomen in case of melena

Situation 5 – Two children were brought to you. One with chest indrawing and the other had diarrhea. The following questions apply:

96. Using Integrated Management and Childhood Illness (IMCI) approach, how would you classify the 1st child?

A. Bronchopneumonia C. Severe pneumonia B. No pneumonia : cough or cold D. Pneumonia

97. The 1st child who is 13 months has fast breathing using IMCI parameters he has:

A. 40 breaths per minute or more B. 50 breaths per minute C. 30 breaths per minute or more D. 60 breaths per minute

98. Nina, the 2nd child has diarrhea for 5 days. There is no blood in the stool. She is irritable, and her eyes are sunken. The nurse offered fluids and and the child drinks eagerly. How would you classify Nina’s illness?

A. Some dehydration C. Severe dehydration B. Dysentery D. No dehydration

99. Nina’s treatment should include the following EXCEPT: A. reassess the child and classify him for dehydration B. for infants under 6 months old who are not breastfed, give 100-200 ml clean water as

well during this period C. Give in the health center the recommended amount of ORS for 4 hours. D. Do not give any other foods to the child for home treatment

100. While on treatment, Nina 18 months old weighed 18 kgs. and her temperature registered at 37 degrees C. Her mother says she developed cough 3 days ago. Nina has no general danger signs. She has 45 breaths/minute, no chest in-drawing, no stridor. How would you classify Nina’s manifestation.

A. No pneumonia C. Pneumonia B. Severe pneumonia D. Bronchopneumonia

SUBMIT THIS TEST BOOKLET TOGETHER WITH THE ANSWER SHEET TO YOUR WATCHERS. BRINGING THE TEST BOOKLET OUT OF THE ROOM WILL BE A GROUND FOR CANCELLATION OF YOUR EXAMINATION.

***END***

Page 28: NLE Compilation 1

28

NURSING PRACTICE III SET A ________________________________________________________________________ NURSING PRACTICE III – Foundation of PROFESSIONAL Nursing Practice GENERAL INSTRUCTIONS:

11. This test booklet contains 100 test questions. 12. Read INSTRUCTIONS TO EXAMINEES printed on your answer sheet. 13. Shade only one (1) box for each question on your answer sheets. Two or more boxes shaded

will invalidate your answer. 14. AVOID ERASURES. 15. This is PRC property. Unauthorized possession, reproduction, and/or sale of this test is

punishable by law. Per RA 8981. ________________________________________________________________________ INSTRUCTIONS:

7. Detach one (1) answer sheet from the bottom of your Examinee ID/Answer Sheet Set. 8. Write the subject title “Nursing Practice III” on the box provided. 9. Shade Set Box “A” on your answer sheet if your test booklet is Set A; Set Box “B” if your test

booklet is Set B. MULTIPLE CHOICE 5. There is an order of Central Venus pressure (CVP) reading. As a nurse that this is a measure of observing signs of:

A. hypoxia C. hypothermia B. hypovolemia D. hypoxemia

6. Pulmonary edema is a potential danger that we nurses should monitor in post pneumonectomy. This is usually due to:

A. extreme temperature

B. liberal fluid intake C. rapid infusion of IV fluids D. fluid retention due to prolonged bed rest

Situation 2-The PRC regulates the practice of 42 professions in the Philippines. 6. What is the basic requirement of the state for a nurse to practice her profession? A. Willingness to practice the profession B. A BSN degree C. A nursing license D. An NCLEX and CGFNS passer 7. The code of Good Governance for the professions in the Philippines shall be adapted by: A. all registered professionals C. all professionals B. all Filipino professionals D. all registered nurses 8. The standardized guidelines and procedures for the implementation of Continuing Professional Education (CPE) for all professional. Resolution Number 2004-179 provides that the total CPE credit units for registered professionals with baccalaureate degree should be: A. 20 credit units per year C. 60 credit units for 3 years B. 30 credit units for 3 years D. 10 credit units required

Page 29: NLE Compilation 1

29

9. The board of Nursing is vested with power to issue, suspend or revoke for cause the: A. certificate of Good Moral Character C. certificate of Registration B. certificate of Practice D. certificate of Employment 10. RA 7193 stipulates the removal examination of the nurse licensure examination shall be taken:

A. Within 3 years after the last failed examination B. Anytime the examinee wants to take the examination C. Within 2 years after the last failed examination D. Within the same year after the filed examination

Situation 3 - Pain is always associated to surgery 11. As a surgical nurse, which of the following nursing intervention will allay anxiety and pain among? surgical patients?

A. Asses the client for concerns especially those that can potentially cause pain B. Verify that the operated permit is signed C. Discourage the client from discussing the details of the surgical procedure D. Ensure safety of client while in surgery

12. Rhizotomy is a condition surgical procedure to manage those that can potentially cause pain. What is the crucial in determining a good candidate for rhizotomy? A. Pain which is resistant to non-pharmacologic for 6 month B. Pain which is resistant to pharmacologic protocol for 12 months C. Local pain with no radiating pain or signs of nervous compassion D. Deep pain with obvious signs of peripheral nerve damage 13. Which of the following would be the nurse’s appropriate response to a crying female client scheduled for emergency surgery who is verbalizing fear of pain but afraid to go to sleep? A. Let her cry and tell significant other to stand by. B. Squeeze her hand and assure her that there will b no pain at all because she will be given anesthesia. C. Stand by her side and quietly ask her to describe her feelings. D. Check her name tag and request anesthesiologist to sedate client 14. Which of the following clients statement indicates that he understands the nurse’s instruction about postoperative wound pain?

A. “I shall call the nurse when my wound itches and smells”. B. “I shall expect slight pain and discomfort from the surgical incision”. C. “I should call my doctor if my wound has no drainage and intact”. D. “I should not touch my surgical wound”

15. What do you think is an important responsibility related to pain that is subjective in nature?

A. Divert attention of client in pain B. Leave the patient alone while in pain C. Believe what the patient says about the pain D. Assume responsibility to eliminate pain as described by the client

Situation 4-Nurse’s attitudes toward the pain influence the way they perceive and interact with clients in pain. 16. Nurses should be aware that older adults are at risk of underrated pain. Nursing assessment and management of pin should address the following beliefs EXCEPT:

Page 30: NLE Compilation 1

30

A. Older patients seldom tend to report pain than the younger ones B. Pain is a sign of weakness C. Older patients do not believe in analgesics, they are tolerant D. Complaining of pain will lead to being labeled a ‘bad’ patient

17. Nurses should understand that when a client responds favorably to a placebo, it is known as the ‘placebo effect’. Placebos do not indicate whether or not a client has:

A. Conscience C. Disease B. Real pain D. Drug tolerance

18. You are the nurse in the pain clinic where you have client who has difficulty specifying the location of pain. How can you assist such client?

A. The pain is vague B. By charting-it hurts all over C. Identify the absence and presence of pain D. As the client to point to the painful are by just one finger

19. What symptom, more distressing than pain, should the nurse monitor when giving opioids especially among elderly clients who are in pain? A. Forgetfulness C. Drowsiness B. Constipation D. Allergic reactions like pruritis 20. Physical dependence occurs in anyone who takes opioods over a period of time. What do you tell a mother of a ‘dependent’ when asked for advice?

A. Start another drug and slowly lessen the opioid dosage B. Indulge in recreational outdoor activities C. Isolate opioid dependent to a restful resort D. Instruct slow tapering of the drug dosage and alleviate physical withdrawal

symptoms Situation 5- As a perioperative nurse, you are aware of the correct processing methods for preparing instruments and other devices for patient use to prevent infection. 21. Items that enter sterile tissue or vascular system are categorized as critical items and should be: A. Clean C. Sterilized B. Decontaminated D. Disinfected 22. As an OR nurse, what are your foremost considerations for selecting chemical agents for disinfection?

A. Material compatibility and efficiency B. Odor and availability C. Cost and duration of disinfection process D. Duration of disinfection and efficiency

23. Before you use a disinfected instrument it is essential that you:

A. Rinse with tap water followed by alcohol B. Wrap the instrument with sterile water C. Dry the instrument thoroughly D. Rinse with sterile water

24. You have a critical heat labile instrument to sterilize and are considering touse high level disinfectant. What should you do?

A. Cover the soaking vessel to contain the vapor B. Double the amount of high level disinfectant C. Test the potency of the high level disinfectant D. Prolong the exposure time according to manufacturer’s direction

Page 31: NLE Compilation 1

31

25. As a nurse, you know that intact skin acts as an effective barrier to most microorganisms. Therefore, items that come in contact with the intact skin should be: A. Disinfected C. Clean B. Sterile D. Alcoholized Situation 6-The OR is divided into three zones to control traffic flow and contamination 26. What OR attires are worn in the restricted area?

A. Scrub suit, OR shoes, head cap B. Head cap, scrub suit, mask, OR shoes C. Mask, OR shoes, scrub suit D. Cap, mask, gloves, shoes

33. Nursing intervention for a patient on low dose IV insulin therapy includes the following, EXCEPT:

A. Elevation of serum ketones to monitor ketosis B. Vital signs including BP C. Estimate serum potassium D. Elevation of blood glucose levels

34. The doctor ordered to incorporate 1000”u” insulin to the remaining on going IV. The strength is 500 /ml. How much should you incorporate into the IV solution? A. 10 ml C. 0.5 ml B. 2 ml D. 5 ml 35. Multiple vial-dose-insulin when in use should be A. kept at room temperature C. kept in narcotic cabinet B. kept in the refrigerator D. store in the freezer Situation 8-Collaborative planning is essential if nursing and health care are to be made available to all people. 36. Perioperative examples of collaboration are the following EXCEPT:

A. Communicate with other members of the health profession to improve the integrity

B. Communicate with health officials the incidence of Hepatitis B among OR personnel

C. Collaboration with other OR personnel regarding the practices of surgeons collecting exorbitant professional fees

D. Collaborate with DOH regarding disposal or specimens 37. The nurses collaborate with other members of the health profession to improve the integrity of the hospital working environment the following ways EXCEPT

A. Joining barangay health club projects B. Joining the Mayo Uno Labor Union C. Joining labor day rally to increase wages of healthcare workers and improve

dilapidated health centers D. Affiliating with The Healthcare Alliance

38. An example of collaborating effort on public service particularly during summer is: A. Boto mo, Ipatrol mo C. Clean and Green B. Operation Linis D. Operation Tuli

Page 32: NLE Compilation 1

32

39. When does a nurse reject the interdependence of providers and patients in achieving access to health care?

A. “Our hospital does not honor visiting doctors” B. When the nurse replies to the client’s relative “You have the best doctor in town” C. When the nurse communicates to the attending physician the desire of the patient

to be seen by a urologist D. “The doctor is not on duty today”

40. Individual patients and society as a whole benefit from nursing participation in decisions made about health care. This is exemplified in:

A. Supporting political candidates that advance nursing care issues B. Bringing the NCLEX in Philippines C. Supporting the proliferation of colleges of nursing in the country D. Following the decision of CGFNS to retake Test III and IV to validate the visa

screen for the U.S. Situation 9-pain management is not limited to pharmacological means: 41. Ronald one of your clients who is being worked out for AIDS tells you that he has been using acupuncture to help with his pain. You questioned his treatment because:

A. Acupuncture uses needles to stimulate certain points on the body to treat pain

B. Acupuncture uses variety of herbs and oils from wild plants C. Acupuncture uses manipulation of the skeletal muscles to relieve stress and pain D. Acupuncture uses pressure from the fingers and hands to stimulate body

responses 42. Your younger brother came home with right black eye. He asked you for an eye ointment to relieve the pain and swelling. You should offer:

A. ice pack over the right eye B. hot compress over the right eye C. tetracycline ophthalmic ointment D. ice cold drinks

45. Menstrual pain and discomfort account for absences in schools and offices. A non- pharmacological remedy for menstrual pain is:

A. regular bowel movement B. knee-chest exercise before menstruation and hot water bag application

over lower abdomen during onset C. warm shower during onset of menstrual period D. diet restriction on fatty foods and liberal fluid intake

Situation 10 – One learns by doing especially when you practice the best methods. 46. Which action by a new nurse signifies a need for further teaching in infection control?

A. The nurse places the side rails the time to an unconscious patient B. The nurse elevates the head of the bed to check the BP. C. The nurse uses her bare hands to change the dressing D. The nurse applies oxygen catheter to the mouth.

47. You are on PM shift and about 5 patients are of discharge. You noted that the orderly was looking through the items of one of the patients. Which action should you pursue?

A. Call the attention of the orderly in private B. Ignore the situation because you are busy C. Report this behavior to the nurse in charge D. Monitor the situation and note whether any other items are reported missing.

48. What appropriate action should you do when you overhear the nursing attendant speaking harshly to an elderly patient?

Page 33: NLE Compilation 1

33

A. Try to explore the interaction with the nursing attendant concerned B. Change the attendant’s assignment C. Initiate a group discussion with all other nursing attendants D. Discuss the matter with the patient’s family

49. You have been in the surgical ward for almost a year and have cared for a number of patients with CVP. Which observation from a colleague would indicate a need for further teaching?

A. The colleague turns the stop-cock to the off position from the IV fluid to the patient

B. The nurse colleague noted the level at the top of the meniscus C. The colleague instructs the clients to perform the valsalva maneuver during

the CVP reading D. The nurse colleague charting medication administration that she has not

yet given 50. You saw one colleague charting medication administration that she has not yet administered. After talking to her, you also report the incident to the charge nurse. The charge nurse should:

A. Require the staff to submit an incident report B. Terminate the nurse C. Charge the erring nurse with dishonesty D. Repot to the Board of Nursing

Situation 11 – You are assigned at the PACU. At 9:30 AM, post-op clients started to be ? in from the OR 51. Which nursing diagnosis has priority among client in the PACU?

A. Acute pain related to discomfort off wound and immobility B. Body image disturbance because of wound dressing and drains. C. Ineffective airway clearance related to general anesthesia. D. Knowledge deficit related to lack of information because patients are all sedated.

52. Which of the following clients at the PACU will demonstrate the effectiveness of ? teaching?

A. The client demonstrated deep breathing, coughing, splintering and leg exercises.

B. The client manifests normal temperature C. The client sleeps well D. The client has good balance I and O

53. Which of the following remark indicates that the client’s relative understood the discharge instruction for wound care?

A. “If the wound is painful, I will say it is normal” B. “It is alright to use adhesive tape over the wound to keep it intact” C. “It is ok for his pet to remain at his bedside to keep him company” D. “I will report any redness or swelling of the wound”

54. You must transfer out a post-op client to her room. What would your instruction to the family include to prevent accidents?

A. Report when the IV infusion is almost finished B. Test the call system if functioning C. Keep the room lights on for 24 hours D. Make sure the side rails are up

55. One of your post-op patients has a temperature of 37.9°C and was shivering. You covered him with a blanket and later took his temperature again and it is now 38.9°C. The nursing student asked you to explain the absence of shivering even if the temperature was higher.

A. The patient is no longer febrile thus he is no longer chilling

Page 34: NLE Compilation 1

34

B. Shivering normally disappears as temperature becomes higher C. The body has reached its new set point thus the absence of shivering D. The patients is feeling better

Situation 12 – Patients with chest tubes can be very challenging to new nurses. 56. The chest tube drainage of Tirso has continuous bubbling in the water seal drainage. After an hour you noticed that the bubbling stops. Which of the following condition is the possible cause of the malfunctioning sealed drainage?

A. A suction being too high C. A tube being too small B. An air leak D. A tension pneumothorax

57. While you were making your endorsement, you found out the chest tube of a client was disconnected. What would be your appropriate action?

A. Assist the client back to his bed and place him on the affected side B. Cover the end of the chest tube while sterilize gauze C. Reconnect the tube to the chest tube system D. Put the end of the chest tube into a cup of sterile normal saline

58. Dr. Reyes asked you to assist him with the removal; of Tirso’s chest tube. You would instruct the client to:

A. Continuously breathe normally during the normal of the chest tube B. Take a deep breath, exhale, and bear down C. Exhale upon actual removal of the tube D. Hold breath until the chest tube is pulled out

59. Chest tube diameter is measured or expressed in:

A. french C. milliliters B. gauge D. inches

60. When transporting clients with chest tube, the system should be:

A. disconnected B. closed C. placed lower than the patient’s chest D. placed between the legs of the client to prevent breakage

Situation 13 – The preoperative nurse collaborates with the client significant others, and healthcare providers. 61. To control environmental hazards in the OR, the nurse collaborates with the following departments EXCEPT:

A. biomedical division C. infection control committee B. chaplaincy services D. pathology department

63. Waste disposal poses a big problem for the hospital. Biological wastes (i.e. amputated limbs) disposal should be coordinated with following agencies EXCEPT:

A. Crematorium C. MMDA B. DOH D. DILG

64. Tess, the PACU nurse, discovered that Malou, who weights 110 lbs prior to surgery, is in severe pain 3 hrs after cholecystectomy. Upon checking the chart, Malou found out that she has an order of Demerol 100 mg I.M. prn for pain. Tess should verify the order with:

A. Nurse supervisor C. Surgeon B. Anesthesiologist D. Intern on duty

Page 35: NLE Compilation 1

35

65. Rosie, 57, who is diabetic is for debridement if incision wound. When the circulating nurse checked the present IV fluid, she found out that there is no insulin incorporated as ordered. What should the circulating nurse do?

A. Double check the doctor’s order and call the attending MD B. Communicate with the ward nurse to verify if insulin was incorporated or not C. Communicate with the client to verify if insulin was incorporated D. Incorporate insulin as ordered.

Situation 14 – Technology and patient’s education has dramatically improved the management of the diabetic client. 66. The current insulin pumps available in the market have the following capability, EXCEPT:

A. Prevent unexpected saving in blood glucose measurements. B. Detect signs and symptoms of hypoglycaemia and hypercalcemia. C. Deliver a pre-meal bolus dose of insulin before each meal. D. Deliver a continuous basal rate of insulin at 5.0 units to 2.0 units per hour

67. Discharge plan of diabetic clients include injection-site-rotation. You should emphasize that the space between sites should be:

A. 6 cm B. 5 cm C. 2.5 cm D. 4 cm

68. It is critical also that a diabetic client should be educated in the possible sites if regular insulin injection. The fastest absorption rate happens at the tissue areas of:

A. Gluteal area B. Deltoid area C. Anterior area D. Abdominal area

69. Self-monitoring of blood glucose (SMBG) is recommended for patient’s use. You will recommend this technology in the following diabetic patients, EXCEPT:

A. Client with proliferative retinopathy B. Unstable diabetes C. Hypoglycemia without warning D. Abdominal renal glucose threshold

70. It is necessary for a diabetic client to exercise regularly. What is the effect of regular exercise to a diabetic client?

A. It burns excess glucose B. It improves insulin utilization and lowers blood glucose C. It lowers glucose, improves insulin utilization; decrease total triglyceride

levels D. It will make you fit and energized

Situation 15- RN’s should always be conscious that the contents in charting are admissible in court as evidence. 71. If there is any deviation from normal practice or procedure e.g. streptomycin was given by IV not IM, this should documented in the:

A. Progress notes B. Incident report C. Nurse’s not D. Patient’s chart

72. The documentation of all nursing activities performed is legally and professionally vital.

Page 36: NLE Compilation 1

36

Which of the following should NOT be included in the patient’s chart?

A. Presence of prosthetoid devices such as dentutes, artificial limbs hearing aid, etc. B. Baseline physical, emotional, and psychosocial data C. Arguments between nurses and residents regarding treatments D. Observed untoward signs and symptoms and interventions including contaminant

intervening factors 73.During your morning rounds, Mr. Tipol, 60 year old widower tried to sit up an instead of holding to the side rail held the IV stand causing the IV bottle to fall and break. You wrote an incident report to show:

A. Document the incident B. Be a part of the patient’s chart C. Present confidential report D. Evidence of the quality of care

74. Erasures, alterations, and additions in medical records and the nurse’s notes can be avoided. The following are some tips on how to do corrections EXCEPT:

A. Cross out blank spaces B. Cross out wrong word or phrase with one or two lines making the crossed out

word discernible C. Insert additions or corrections D. State the reason for any deviation from normal procedure/practice

75. Kathy is one of your patient’s. Her uncle, who is a doctor, wants to read her chart. Your appropriate action would be:

A. Instruct Kathy’s uncle to present a written authorization signed by the patient

B. Refer to the hospital director C. Instruct Kathy’s uncle to present a written request to the Medical Records Section

of the hospital D. Refer to the attending physician

Situation 16- During the month of July, you noticed that there is an incidence of upper respiratory Disorders. 76. One of your cases is with acute pharyngitis. Your nursing management includes the following EXCEPT:

A. Suggest a soft or liquid diet during acute stage B. Encourage liberal amount of cold fruit juices C. Encourage bed rest during stage D. Apply ice collar for sympathetic relief of severe sore throat

77. For nurse to asses an upper respiratory tract infection, you should palpate the following:

A. The ears, eyes, nose, and throat B. Adenoids tonsils and nose C. Nose and throat only D. The tracheal and nasal mucosa including the frontal sinuses

78. Among the patients with upper airway infection, airway clearance can be facilitated by the following EXCEPT:

A. Regularly administering prescribed vasoconstrictive medication B. Decreasing systematic hydration C. Positional drainage D. Humidifying inspired room air

79. A friend asked you some nursing measures of uncomplicated common colds. You will include the following measures EXCEPT:

Page 37: NLE Compilation 1

37

A. Instruct client about symptoms of secondary infections B. Administer prescribed antibiotics C. Teach that the causative virus is contagious even before symptoms appear D. Suggest adequate of fluids and rest

80. The following are your nursing suggestions for a patient with acute or chronic sinusitis EXCEPT:

A. Local heat application to promote drainage B. Consult an ENT surgeon C. Increase humidity D. Advice adequate fluid intake

Situation 17- A specimen is a piece of tissue or body fluid taken from the disease body organ or tissue to aid the health care team in diagnosis and effective treatment. 81. Carmen is suspected to have a left CA. She is scheduled in your room on 5 for frozen section. How will you prepare the specimen for laboratory?

A. Refrigerate and send it along with the day’s specimens B. Send to pathology immediately without soaking solution C. Soak it in NSS D. Soak it in formalin

82. How will you label this specimen? What information was essential in the label?

A. Name of the client, age, sex, hospital number B. Name of the client, age, sex C. Name, age, site, type of specimen, hospital number, doctor D. Name, doctor, type of specimen, hospital number

83. Foreign body the extracted from the body like pins, needles, seeds or bullets are also considered as a specimen. You assisted in the multiple gun-shot wound exploration. During the surgery the specimen to:

A. The department of pathology B. The national bureau of investigation C. The OR head nurse D. Client’s family

84. A post dilation and curettage (D and C) client is for discharge. Follow-up of lab result should be part of the discharge plan. You will instruct the client to follow up result at the:

A. Medical record B. Laboratory C. Doctor’s clinic D. Nurse’s station

85. You are the circulating nurse in OR 2. You have 4 thyroidectomy cases for the day. How do you prevent switching of specimens?

A. Send specimens to laboratory right away after the operation with the proper labels

B. Collect all specimens and send to laboratory at the end of the day C. Label specimen at once D. Prepare 4 specimen vials first thing in the morning

Situation 18- Mr. Santos, 50, is to undergo cystoscopy due to multiple problems like scantly urination, hematuria and dysuria. 86. You are the nurse in charge in Mr. Santos. When asked what are the organs to be examined during cystoscopy, you will enumerate as follows:

A. Urethra, kidney, bladder, urethra

Page 38: NLE Compilation 1

38

B. Urethra, bladder wall, trigone, ureteral opening C. Bladder wall, uterine wall, and urethral opening D. Urethral opening, ureteral opening bladder

87. In the OR, you will position Mr. Santos who is cystoscopy in:

A. Supine B. Lithotomy C. Semi-fowler D. Trendelenburg

88. After cystoscopy, Mr. Santos asked you to explain why there is no incision of any kind. What do you tell him?

A. “Cystoscopy is direct visualization and examination by urologist”. B. “Cystoscopy is done by x-ray visualization of the urinary tract”. C. “Cystoscopy is done by using lasers on the urinary tract”. D. “Cystoscopy is an endoscopic procedure of the urinary tract”.

89. Within 24-48 hours post cystoscopy, it is normal to observe one the following:

A. Pink-tinged urine B. Distended bladder C. Signs of infection D. Prolonged hematuria

90. Leg cramps are NOT uncommon post cystoscopy. Nursing intervention includes:

A. Bed rest B. Warm moist soak C. Early ambulation D. Hot sitz bath

Situation 19- During the surgical procedure, contamination should be confined and contained within the immediate vicinity of the surgical field to prevent the spread of pathogenic microorganisms. 91. The following techniques illustrates the concept “confine and contain” EXCEPT:

A. Contaminated items like sponges are handled using loves B. All blood tissue, and body fluid specimens should be placed in leak-proof

containers C. Surgeons conduct their patient’s rounds in scrub suit D. Prompt cleanup of accidental spills of contaminated debris e.g. blood, body fluids

92. The OR is a restricted area where OR attire is worn. Temperature and humidity are set for patient and personnel safety and reduce bacterial contamination. During the surgery, movement of personnel including the circulating nurse is:

A. Kept to minimum B. Eliminated when possible C. Restricted D. Monitored

93. Sterile is the condition of almost all items, devices or supplies used in the OR for any surgical procedure. Shelf-life of a packaged sterile item is event related and depends on the following, EXCEPT:

A. Type of sterilizer used to sterilize items B. Amount of handling C. The quality of packaging material used D. Storage conditions

94. Precaution recommends that the use of standard personal protective equipment (PPE) to prevent cross contamination. Which is NOT considered a piece of PPE?

A. Cover gown

Page 39: NLE Compilation 1

39

B. Eyewear C. Gloves D. Face shields

95. Traffic patterns in the OR suite should.

A. Prevent unauthorized personnel from entering the OR B. Prevent transmission of pathogenic microorganisms C. Assure that the personnel walk in the same direction D. Allows personnel to move freely between restricted and unrestricted areas

Situation 20- Nokia is so powerful to “connect people” from the continent to continent, all through communication via the cellphone. Other ways of communicating to relay information or instructions exist even in the healthcare setting. 96. An anesthesiologist is preparing to do a spinal anesthesia to a 220 lb, 30 year old athlete she request the circulating nurse to prepare a pink spinal set with another blue set as stand by. What gauge spinal sets will make available in the OR suite?

A. Gauge 16 and 22 B. Gauge 18 and 16 C. Gauge 16 and 20 D. Gauge 5 and 22

97. Medical gases are used a lo9t in the OR. Some gases are used to operate equipment and some are used to administer general anesthesia through inhalation. What is the identifying color of the tank which contains ‘laughing gas’?

A. Yellow B. Green C. Black D. Blue

98. On the traffic light, yellow means “proceed with caution”. In the field of healthcare, where do you discard your used tissue papers?

A. Yellow bin B. Orange bin C. Green bin D. Black bin

99. An instrument tray with black striped autoclave/steam chemical indicator tape communicates that the instrument tray...

A. Is clean B. Is ready for use in surgery C. Is sterile D. Has undergone the sterilization process

100. In health care when lad apron is required in any procedure like orthosurgery, there is danger of exposure to:

A. Water and blood splashes B. Pseudomonas C. Radiation D. Bone fragments

SUBMIT THIS TEST BOOKLET TOGETHER WITH THE ANSWER SHEET TO YOUR WATCHERS. BRINGING THE TEST BOOKLET OUT OF THE ROOM WILL BE A GROUND FOR CANCELLATION OF YOUR EXAMINATION.

Page 40: NLE Compilation 1

40

***END*** NURSING PRACTICE IV SET A ________________________________________________________________________ NURSING PRACTICE IV – Foundation of PROFESSIONAL Nursing Practice GENERAL INSTRUCTIONS:

16. This test booklet contains 100 test questions. 17. Read INSTRUCTIONS TO EXAMINEES printed on your answer sheet. 18. Shade only one (1) box for each question on your answer sheets. Two or more boxes shaded

will invalidate your answer. 19. AVOID ERASURES. 20. This is PRC property. Unauthorized possession, reproduction, and/or sale of this test is

punishable by law. Per RA 8981. ________________________________________________________________________ INSTRUCTIONS:

10. Detach one (1) answer sheet from the bottom of your Examinee ID/Answer Sheet Set . 11. Write the subject title “Nursing Practice IV” on the box provided. 12. Shade Set Box “A” on your answer sheet if your test booklet is Set A; Set Box “B” if your test

booklet is Set B. MULTIPLE CHOICE

101. You are the nurse in an Adult Care Unit. You over0hear one of your co-staff nurse assigned to Aling Josie who is 78 years old say, that if she refuses to take her medications, she will not be given her favorite dessert. You report your co-staff’s behavior as:

A. Battery C. Negligence B. Assault D. Malpractice

102. Jake is in the Post Anesthesia Care Unit follwing a colorectal resection. He has an IV of Dextrose 5% Lactated Ringers Solution. Upon assessment you observe that he is exhibiting sudden onset of crackles in the lungs, moist respiration and tachypnea. Which of the following will you do FIRST?

A. Notify anesthesiologist C. Place on Fowler’s position B. Increase O2 flow rate D. Reduce IV rate

103. As a head nurse of the unit, which of the following sources should you take into consideration when making effective assignments for the next shift?

A. seniority preferences B. recent performance evaluation C. personality traits D. client classification data

104. Four clients injured in an automobile accident enter the emergency department (ED) at the same time and are immediately seen by a triage nurse. As the triage nurse, you would assign the HIGHEST priority to the client with the:

A. severe head injury and no blood pressure B. maxillofacial injury and gurling respirations C. second trimester pregnancy with premature labor D. lumbar spinal cord injury and lower extremity paralysis

Page 41: NLE Compilation 1

41

105. When a nurse volunteers to work in a hospital setting and she commits a mistake, who is legally responsible?

A. volunteer nurse, hospital and the nurse in charge B. the professional organization which the volunteer nurse represents C. hospital D. volunteer nurse because there is no employer employee relationship

106. Daniel with multiple myeloma complains of deep bone pain. As his nurse, which of the following will you do FIRST?

A. Assess bone pain B. Administer prescribed analgesic C. Teach pain relief strategies D. Support position with pillow

107. You are reviewing the laboratory results of Clare who has rheumatoid arthritis. Which laboratory result should you expect to find?

A. Increased platelet count B. Altered blood urea nitrogen (BUN) and creatinine levels C. Electrolyte imbalance D. Elevated erythrocyte sedimentation rate (ESR)

108. Mrs. Paras is receiving total parenteral nutrition (TPN). If you will evaluate her nutritional status, which of the following indicators will tell you that TPN was effective?

A. laboratory work up B. adequate hydration C. weight gain D. diminish episode of nausea and vomiting

109. While Jayvee, a burn patient is being transferred from the burn unit to the operating

room, the IV bottle fell on Jayvee’s head. He sustained a laceration on his forehead. The nurse was proven guilty of negligence. Which of the following did the nurse fail to do?

A. Hold the IV bottle B. Check the IV stand C. Place the IV stand on the foot part of the stretcher D. Restrain Jayvee

110. While Mrs. Enriquez is receiving chemotherapy which of the following will you include in the plan of care to address her nutritional needs?

A. administer Compazine before meals B. enrich diet with red meats C. serve hot soup and food D. increase the amount of spice in the diet

111. Nurses working in the 35 bed Female Medical Unit were noted to implement new and innovative client care activities long before other units in the hospital. Which of the following leadership characteristics exhibited by the nurse manager best describes this strength?

A. Communication skills C. Vision and passion B. Knowledge and skills D. Interpersonal abilities

112. Olga is receiving D5W 1 liter regulated at 30 drops/min to be consumed in 8 hrs. It was started at 8am. At 10am, her relative informed you that the bottle is empty. Which of the following will you do first?

A. refer to nurse manager B. assess Olga and check level of fluid left in the bottle

Page 42: NLE Compilation 1

42

C. discontinue IV and assess Olga D. replace IV fluid with prescribed follow-up

113. A research study found out that 60% of patients complains were due to delayed

responses of nurses in the emergency department. Which of the following measurement of data was used in this study?

A. Measures of variability C. Frequency distribution B. Measures of central tendency D. Inferential statistics

114. During a meal, a client with hepatitis B dislodges her IV line and bleeds on the surface of the over-the-bed table. It would be most appropriate for the nurse to instruct a housekeeper to clean the table with:

A. Alcohol C. ammonia B. Acetone D. bleach

115. Nino is being treated with radiation therapy. What should be included in the plan of care to minimize skin damage from the radiation therapy?

A. Cover the areas with thick clothing materials B. Apply a heating pad to the site C. Wash skin with water after the therapy D. Avoid applying creams and powders to the area

116. You are assigned to the following patients. Which of the following patients is most at risk for metabolic alkalosis?

A. Grace, 30 year old post surgical patient who has continuous nasogastric

suction B. Rachel a 55 year old who has just experienced a stroke C. Helen, 70 year old with altered level of consciousness who is unable to access

water freely D. Mary Jane a 2 year old infant receiving isotonic sodium chloride IV solution

117. Studies have shown that the highest incidence of Hodgkin’s disease is common among young adults. Juana, 20 years old approaches you and tells you “I am worried about the mass on my neck”. What should you do as a nurse?

A. Tell her there is nothing to worry if it does not bother her B. Palpate Juana’s neck and explain the possible cause C. Tell her Hodgkin’s disease is common among young adults like her D. Tell her to see a doctor

118. As a nurse, you accidentally administer 40 mg of Propanolol (Inderal) to a client instead of 10 mg. Although the client exhibits no adverse reactions to the larger dose, you should:

A. complete an incident report B. call the hospital attorney C. inform the client’s family D. do nothing because the client’s condition is stable

119. You are the nurse manager of the Medical Unit. Which of the following is a priority for

you to consider when planning for the care of a group of clients utilizing evidence-based practice?

A. Client’s care is planned based on the nurse’s clinical expertise and latest

research findings B. Standardized care plans are used on all of the nurse’s clients. C. Standards of care are developed by the hospital nursing service and should be

followed

Page 43: NLE Compilation 1

43

D. Client’s needs are assessed and individualized care plan are developed for each client.

120. Because of increase incidents of medication error due to wrong transcription of physician medication orders by the nurse, a tertiary hospital utilized a computerized medication order system. Which of the following procedures may be done through the said system?

A. Correct errors in the physician medication order B. Eliminate drug interaction C. Provide a list of drugs with their generic name D. Document drug administration

SITUATIONAL Situation 1 – P. Cruz, 65 years old, was admitted in the Telemetry because of signs and symptoms of acute myocardial infarction. You are expected to recognize electrocardiographic readings on the cardiac monitor.

121. Which of the following appear abnormal on an EKG when ischemia and injury occur in the myocardium?

A. QRS interval C. P wave B. ST segment D. PR interval

122. From an ECG reading, a QRS complex represents: A. Venticular depolarization C. End of ventricular

depolarization B. Ventricular repolarization D. Atrial depolarization

123. Which of the following represents ventricular repolarization?

A. T wave C. QRS complex B. ST segment D. PR interval

124. It is important that the nurse measures intervals of QRS complex. Which of the following represent the normal interval of QRS complex?

A. greater than .20 sec C. .10 sec B. .20 sec D. .12 sec to .20 sec

125. Later in the acute phase of Myocardial Infarction, which of the following typically appears as the first sign of tissue death?

A. ST segment suppression C. Prolonged PR interval B. Short T wave D. Pathologic Q wave

Situation 2 – To be able to help our clients with their psychological concerns, we have to explore how they view themselves and their body image.

126. When assessing patient’s body image, which of the following would be most important to ask?

A. What are your hobbies? B. What kind of work did you do prior to this illness? C. Are your living accommodation all on one level? D. What kind of food do you like?

127. The most appropriate nursing intervention to facilitate client’s acceptance of a change

in body image would be to:

A. encourage dependence B. establish a therapeutic relationship

Page 44: NLE Compilation 1

44

C. joke with the client D. establish a social relationship

128. Which of the following responses would indicate that the client is beginning to accept change in his/her body image?

A. Feeling of the dressing over the incisional site B. Stating he/she is too tired to have visitors C. Discussing his.her stamp collection with the nurse D. Showing no interest in the dressing change

129. It is important for you to remember, that a sudden change in body image would occur from:

A. Weight gain B. Side effects of medication on skin C. Radiation treatment of the breast D. Surgical removal of an eyeball

130. Before you can help Lorna accept a change in body image you must FIRST:

A. be in agreement with the philosophy of therapy for the client B. become aware of her own attitude toward mutilation and disfigurement C. be aware of the attitudes and feelings of the client and her family D. accept the fact that a person can live with a body part missing

Situation 3 – Radiation therapy is another modality of cancer management. With emphasis on multidisciplinary management you have important responsibilities as nurse.

131. Albert is receiving external radiation therapy and he complains of fatigue and malaise. Which of the following nursing interventions would be most helpful for Albert?

A. Tell him that sometimes these feelings can be psychogenic B. Refer him to the physician C. Reassure him that these feelings are normal D. Help him plan his activities

132. Immediately following the radiation teletherapy, Albert is

A. considered radioactive for 24 hrs B. given a complete bath C. placed on isolation for 6 hours D. free from radiation

133. Albert is admitted with a radiation induced thrombocytopenia. As a nurse you should observe the following symptoms:

A. petechiae, ecchymosis, epistaxis B. weakness, easy fatigability, pallor C. headache, dizziness, blurred vision D. severe sore throat, bacteremia, hepatomegaly

134. What nursing diagnosis should be of highest priority?

A. Knowledge deficit regarding thrombocytopenia precautions B. Activity intolerance C. Impaired tissue integrity D. Ineffective tissue perfusion, peripheral, cerebral, cardiovascular, gastrointestinal,

renal

135. What intervention should you include in your care plan?

Page 45: NLE Compilation 1

45

A. Inspect his skin for petechiae, bruising, GI bleeding regularly B. Place Albert on strict isolation precaution C. Provide rest in between activities D. Administer antipyretics if his temperature exceeds 38C

Situation 4 – Andrea is admitted to the ER following an assault where she was hit in the face and head. She was brought to the ER by a police woman. Emergency measures were started.

136. As Andrea’s nurse, what will be your priority interventions

A. Insert an intravenous catheter B. Insert an oral or nasopharyngeal airway C. Obtain arterial blood gases D. Give 100% oxygen by mask

137. Andrea’s arterial blood gases reflect respiratory acidosis. This is most likely related to:

A. Partially obstructed airway B. Ineffective breathing pattern C. Head injury D. Pain

138. Andrea loses consciousness. You should prepare for which of the following FIRST?

A. Placement of a nasogastric tube B. Placement of a second IV line C. Endotracheal intubation or surgical airway placement D. CT scan of the head

139. Andrea’s physician gives an order of Mannitol 0.25 g/kg IV bolus for increased ICP. This is given to:

A. promote cerebral-tissue fluid movement B. promote renal perfusion C. correct acid-base imbalances D. enhance renal excretion of drugs

140. As Andrea’s nurse your goal is to prevent increased intracranial pressure (ICP). Which of the following independent nursing interventions nursing interventions is NOT suited for her?

A. Do oropharyngeal suction every 15 minutes to prevent pulmonary aspiration B. Keep head of bed 30-45 degrees elevated C. Maintain Andrea’s head in straight alignment and prevent hip flexion D. Prevent constipation and increases in intra-abdominal pressure

Situation 5 – Specific surgical interventions may be done when lung cancer is detected early. You have “important peri-operative” responsibilities in caring for patients with lung cancer.

141. Horace underwent lobectomy. Which of the following is the purpose of Horace’s closed chest drainage post lobectomy?

A. Facilitation of coughing B. Promotion of wound healing C. Expansion of the remaining lung D. Prevention of mediastinal shift

142. Which of the following observations indicates that the closed chest drainage system is

functioning properly?

A. Absence of bubbling in the suction-control bottle B. The fluctuating movement of fluid in the long tube of the water-seal bottle

during inspiration C. Intermittent bubbling through the long tube of the suction control bottle

Page 46: NLE Compilation 1

46

D. Less than 25 ml drainage in the drainage bottle

143. Following lobectomy, you can best help Horace to reduce pain during deep breathing and coughing exercises by:

A. placing him on his operative side during exercises B. splinting his chest with both hands during the exercises C. administering the prescribed analgesic immediately prior to exercises D. providing rest for six hours before exercises

144. Peter underwent pneumonectomy. During the immediate postoperative period, deep tracheal suction should be done with extreme caution because:

A. Peter will not be able to tolerate coughing B. The tracheobrachial trees are dry C. The remaining normal lung needs minimal stimulation D. The bronchial suture line may be traumatized

145. On which of the following positions should you place Peter who just underwent pneumonectomy?

A. Prone position B. On his abdomen or on the side opposite the surgery C. On his back or on the side of surgery D. Any position is acceptable

Situation 6 – As a nurse you should be able to address problems and discomforts experienced by the acutely ill older persons.

146. Pain in the older persons require careful assessment because they:

A. Are expected to experience chronic pain B. Experienced reduce sensory perception C. Have increased sensory perception D. Have a decreased pain threshold

147. Administration of analgesic to the older persons requires careful patient assessment because older people:

A. have increased hepatic, renal and gastrointestinal functions B. mobilize drug more rapidly C. have increased sensory perception D. are more sensitive to drugs

148. The older person is at higher risk for incontinence because of:

A. increased glomerular filtration C. diuretic use B. decreased bladder capacity D. dilated urethra

149. The most dependable cause of infection in the older person is

A. change in mental status B. fever C. decreased breath sounds with crackles D. pain

150. Your priorities when caring for the older person who sustained traumatic injuries include:

A. circulation, airway, breathing B. airway, breathing disability (neurologic) C. airways, breathing, circulation D. disability (neurologic), airway, breathing

Page 47: NLE Compilation 1

47

Situation 7 – Mang Felix, a 79 year old man who is brought to the Surgical Unit from PACU after a transurethral resection. You are assigned to receive him. You noted that he has a 3-way indwelling urinary catheter for continuous fast drip bladder irrigation which is connected to a straight drainage.

151. Immediately after surgery, what would you expect his urine to be?

A. Light yellow C. Bright red B. Amber D. Pinkish to red

152. The purpose of the continuous bladder irrigation is to:

A. allow continuous monitoring of the fluid output status B. provide continuous flushing of clots and debris from the bladder C. allow for proper exchange of electrolytes and fluid D. ensure accurate monitoring of intake and output

153. Mang Felix informs you that he feels som discomfort on the hypogastric area and he has to void. What will be your most appropriate action?

A. Remove his catheter then allow him to void on his own B. Irrigate his catheter C. tell him to “Go ahead and void. You have an indwelling catheter.” D. assess color and rate of outflow, if there is a change refer to urologist for

possible irrigation.

154. You decided to check on Mang Felix’s IV fluid infusion. You noted a change in flow rate, pallor and coldness around the insertion site. What is your assessment finding?

A. Phlebitis B. Infiltration to subcutaneous tissue C. Pyrogenic reaction D. Air embolism

155. Knowing that proper documentation of assessment findings and interventions are important responsibilities of the nurse during first post operative day, which of the following is the LEAST relevant to document in the case of Mang Felix?

A. Chest pain and vital signs B. Intravenous infusion rate C. Amount, color, and consistency of bladder irrigation drainage D. Activities of daily living started

Situation 8 – Many hospitals form bioethical review committees to ensure better quality of life of patients. You are invited by the nursing service department to participate in their bioethical review committee. You are expected to know the purpose and apply bioethical principles.

156. Which of the following is the purpose of the ethical review committee?

A. Promote implementation of general standards B. Enhance health care provider’s liability C. Increase individuals responsibility for decision making D. Decrease public scrutiny of health care provider’s action

157. Daria who is admitted to the hospital with autoimmune thrombocytopenia and a platelet count of 20,000/æL develops epistaxis and melena. Treatment with corticosteroids and immunoglobulin has not been successful. Her physician recommended splenectomy. Daria states “I don’t need surgery. This will go away on its own.” In considering your response to Daria, you must depend on the ethical principle of:

A. Beneficence C. Autonomy

Page 48: NLE Compilation 1

48

B. Justice D. Advocacy

158. Zorayda is terminally ill and is experiencing severe pain. She as bone and liver metastasis. She has been on morphine for several months now. Zorayda is aware that they are having financial problems. She decides to sign a DNR form. What ethical principle did Zorayda and her family utilize as basis for their decision to sign a DNR.

A. Justice C. Beneficence B. Autonomy D. Advocacy

159. Tricia, a staff nurse working in a cancer unit, is considered a role model not only by her

colleagues but also by her patients. She goes out of her way to help other. She is very active in their professional organization and she practices what she teaches. What ethical principles is she practicing?

A. Beneficence C. Advocacy B. Autonomy D. Justice

160. You are commuting to work riding the LRT. An older person collapsed and nobody seem to notice her. The security guard tried to make her sit down but she remained unconscious. You saw what happened and you decided to help. With help, you brought the patient to the nearest hospital. You learned later that woman was diabetic. She was on her way to the diabetes clinic to have her fasting blood sugar tested. She developed hypoglycemia. You were able to save a life. You felt good. What principle was applied?

A. Advocacy C. Justice B. Beneficence D. Autonomy

Situation 9 – Ensuring safety before, during and after a diagnostic procedure is an important responsibility of the nurse.

161. To help Fernan better tolerate the bronchoscopy, you should instruct him to practice

which of the following prior to the procedure?

A. clenching his fist every 2 minutes B. breathing in and out through the nose with his mouth open C. tensing the shoulder muscles while lying on his back D. holding his breath periodically for 30 seconds

162. Following a bronchoscopy, which of the following complains to Fernan should be noted as a possible complication:

A. nausea and vomiting B. shortness of breath and laryngeal stridor C. blood tinged sputum and coughing D. sore throat and hoarseness

163. Immediately after bronchoscopy, you instructed Fernan to:

A. exercise the neck muscles B. breathe deeply C. retrain from coughing and talking D. clear his throat

164. Thoracentesis may be performed for cytologic study of pleural fluid. As a nurse your most important function during the procedure is to:

A. keep the sterile equipment from contamination B. assist the physician C. open and close the three-way stopcock D. observe the patient’s vital signs

165. Right after thoracentesis, which of the following is most appropriate intervention?

Page 49: NLE Compilation 1

49

A. instruct the patient not to cough or deep breathe for two hours B. observe for symptoms of tightness of chest for bleeding C. place an ice pack to the puncture site D. remove the dressing to check for bleeding

Situation 10 – As a nurse you are expected to be competent in utilizing the nursing process in the care of your clients.

166. Getty is receiving chemotherapy for cancer. You review Getty’s laboratory report and note that he has anemia. To which nursing diagnosis should you give the highest priority?

A. Activity intolerance B. Impaired oral mucous membrane C. Impaired tissue perfusion, cerebral, cardiovascular, gastrointestinal D. Impaired tissue integrity

167. An immediate objective for nursing care of an overweight mildly hypertensive client with ureteral colic and hematuria is to decrease:

A. Hypertension C. hematuria B. Pain D. weight

168. A difficult problem to deal with when caring for a patient with a partial-thickness burns sustained 3 days ago is:

A. alteration in body image C. frequent dressing change B. maintenance of sterility D. severe pain

169. Which outcome criterion would be most appropriate for a client with a nursing diagnosis of ineffective airway clearance?

A. Continued use of oxygen when necessary B. Breath sounds clear on auscultation C. Respiratory rate of 24/min D. Presence of congestion

170. Which assessment would be most supportive of the nursing diagnosis, impaired skin integrity related to purulent wound drainage.

A. Heart rate of 88 beats/min C. Oral temperature of 38.8 deg C B. Dry and intact wound dressing D. Wound healing by first intention

Situation 11 – Nurses have important responsibilities when caring for hospitalized acutely ill patients.

171. Domingo, 80 years old diabetic and hypertensive is admitted in the private ward for

degenerative neurological changes. His physician was considering dementia. Side rails were placed to ensure that he will not fall from bed. At 2:00 AM, the call light at his room was on. You came in and saw Domingo slumped on the floor moaning. His daughter told you that he got out of bed to go to the toilet. He climbed over the side rail but his foot got caught in the beddings. He has an open wound on his forehead. Which among the following will you do FIRST?

A. Transfer him to bed B. Apply restraints C. Ensure airway, breathing, circulation D. Call his physician

Page 50: NLE Compilation 1

50

172. Aimee has chest pain and decides to take nitroglycerine en route to the hospital. Based on the ECG obtained on admission at the ER and clinical findings, the physician gave a diagnosis of myocardial infarction (MI) and prescribed IV morphine to relieve continuing pain. A primary goal of nursing care for Aimee is to recognize life-threatening complications of MI. As Aimee’s nurse, you have to anticipate occurrence of complications. Take note that the major cause of death after an MI is:

A. Cardiac arrhythmias C. Cardiogenic shock B. Heart failure D. Pulmonary embolism 173. The cardiac monitor indicates that Cedric’s heart rate has increased to 150 beats per

minute. Shortly after this increase, you notice Cedric is in ventricular tachycardia. after reporting this to the physician, you anticipate that the physician will order.

A. intracardiac epinephrine B. insertion of a pacemaker C. bolus of Lidocaine D. manual cardiopulmonary resuscitation

174. Hermie with a left-sided heart failure complains of increasing shortness of breath and is agitated and coughing up of pink-tinged foamy sputum. You should recognize this as signs and symptoms of:

A. cardiogenic shock C. acute pulmonary edema B. right-sided heart failure D. pneumonia

175. You are caring for Lulu has acute pulmonary edema. To immediate promote oxygenation and relief of dyspnea, you should first:

A. perform chest physiotherapy B. have her take deep breaths and cough C. place Lulu on high fowler’s position D. administer oxygen

Situation 12 – Acute respiratory distress is life threatening. Your presence and critical decision making as a nurse are important.

176. Frank is admitted to the Intensive Care Unit with a diagnosis of acute respiratory distress syndrome. When assessing Frank you would expect to find:

A. facilitate Frank’s verbal communication B. maintain sterility of the ventilation system C. assess his response to the equipment D. prepare him for emergency surgery E. Dyspnea, Flaring of the Nares

177. Frank’s respiratory status necessitates endotracheal intubation and positive pressure ventilation. Your most immediate nursing intervention for Frank at this time would be to:

A. facilitate Frank’s verbal communication B. maintain sterility of the ventilation system C. assess his response to the equipment D. prepare him for emergency surgery

178. Tiger with a pulmonary embolus is intubated and placed on mechanical ventilation. When suctioning the endotracheal tube, you should:

A. apply suction while inserting the catheter B. use short and jabbing movements of the catheter to loosen secretions C. hyperoxygenate with 100% oxygen before and after suctioning D. suction two to three timed in quick succession to remove all secretions

Page 51: NLE Compilation 1

51

179. As a nurse, you should observe Bernard, who has a restrictive airway disease, for early indications of respiratory acidosis, which include:

A. light-headedness C. bradycardia B. bradypnea D. restlessness 180. The physician orders low concentration oxygen to be given continuously for Kenneth

who has a chronic obstructive pulmonary disease to prevent:

A. an excessive drying of the respiratory mucosa B. depression of the respiratory center C. rupture of emphysematous bullae D. a decrease in red blood cell formation

Situation 13 – As a nurse you have to be prepared to care for patients receiving blood transfusion. The physician has ordered 3 units of whole blood to be transfused to Diego following following a repair of a dissecting aneurysm of the aorta.

181. You are preparing a unit of whole blood for transfusion. From the time you obtain it

from the blood bank how long should you infuse it?

A. 4 hours C. 2 hours B. 1 hour D. 6 hours

182. What should you do FIRST before you administer blood transfusion?

A. Check IV site and use appropriate BT set and needle B. Verify physician’s order

C. Verify client identity and blood product blood product, serial number, blood type, cross matching results, expiration date with another nurse

D. Verify client identity and blood product, serial number, blood type, cross matching results, expiration date

183. As Diego’s nurse what will you do after the transfusion has been started?

A. Discontinue the primary IV of Dextrose 5% water B. Stay with Diego for 15 minutes to note for any possible BT reaction C. Check his vital signs every 15 minutes D. Add the total amount of the blood to be transfused to the intake and output

184. Diego is undergoing blood transfusion of the first unit. The earliest signs of transfusion reactions are:

A. oliguria and jaundice B. urticaria and wheezing C. hypertension and flushing D. headache, chills, fever

185. In case Diego will experience an acute hemolytic reaction, what will be your priority intervention?

A. Immediately stop the blood transfusion, infuse dextrose 5% in water and call the

physician B. Slow the blood transfusion and monitor the patient closely C. Immediately stop the blood transfusion, notify the blood bank and administer

antihistamines D. Immediately stop the blood transfusion, infuse normal saline solution, call

the physician, notify the blood bank

Page 52: NLE Compilation 1

52

Situation 14 – Based on studies of nurses working in special units like the intensive care unit and coronary care unit, it is important for nurses to gather as much information to be able to address their needs for nursing care.

186. Critically ill patients frequently complain about which of the following when

hospitalized?

A. Hospital food C. Lack of privacy B. Lack of blankets D. Inadequate nursing staff

187. Who of the following is at greatest risk of developing sensory problem?

A. Female patient C. Transplant patient B. Adoloscent D. Unresponsive patient

188. Which of the following factors may inhibit learning in critically ill patients?

A. gender B. medication C. educational level D. previous knowledge of illness

189. Which of the following statements does not apply to critically ill patients?

A. Majority need extensive rehabilitation B. All have been hospitalized previously C. Are physically unstable D. Most have chronic illness

190. Families of critically ill patients desire which of the following needs to be met first by the nurse?

A. Provision of comfortable space B. Emotional support C. Updated information on client’s status D. Spiritual counseling

Situation 15 – Pain is the most common reasons why people consult their physicians. It is now regarded as the 5th vital sign. This strategy is used to give emphasis on how pain should be managed. You have collaboration as well as independent nursing interventions for pain.

191. The WHO Analgesic ladder provides the health professional with:

A. pharmacologic and nonpharmacologic pain management choices B. general pain management choices based on level of pain C. nonpharmacologic interventions based on level of pain D. specific pain management choices based on severity of pain

192. As a nurse caring for patients in pain, you should evaluate for opioid side effects which include the following EXCEPT:

A. physical dependence C. Respiratory depression B. pruritus D. constipation

193. Which of the following statements about cancer pain is NOT TRUE:

A. undertreatment of pain is often due to a clinician’s failure to evaluate the severity of the client’s problem

B. adjuvant medication such as steroids, anticonvulsants, non-steroidal anti-inflammatory drugs enhance pain perception

C. opioids are drug of choice for severe pain

Page 53: NLE Compilation 1

53

D. pain associated with cancer and the terminal phase of the disease occurs in majority of patients

194. Jack has been on morphine on a regular basis for several weeks. He is now

complaining that the usual dose he has been receiving is no longer relieving his pain as effectively. Assuming that nothing has changed in his condition, you would suspect that Jack is:

A. becoming psychologically dependent B. needing to have morphine discontinued C. developing tolerance to morphine D. exaggerating his level of pain

195. The guidelines for choosing appropriate nonpharmacologic interventions for pain include all of the following EXCEPT:

A. pain problem identification B. type of opioid being used C. skill of health professional D. effectiveness for patient

Situation 16 – The nurse’s accurate assessment is very crucial in preventing complication during the severe post burn period.

196. Dino sustained circumferential thermal burns of the left upper extremity and chest. You

noted that pulse could not be appreciated in his injured extremity. Which of the following will you do FIRST?

A. Elevate the injured extremity to increase blood flow to the heart B. Remove the dead tissues which impede circulation C. Try to take the pulse in the uninjured extremity D. Notify the physician immediately as this requires emergency intervention

197. While unloading containers with chemicals from a truck, Mark accidentally spilled the

whole can of corrosive chemicals all over his body. Which of the following would you consider as the priority intervention in the emergency management of Mark?

A. Maintain a patent airway B. Wash the chemical off with cool water C. Assess for associated injuries D. Remove all clothing containing the chemical

198. You are assigned in the Burn Unit and you are going to evaluate the status of Raymond who sustained a burn injury 12 hours ago and has a urinary output of 200 ml since the injury. Which of the following will you do FIRST?

A. Increase the rate of the IV fluid B. Administer the prescribed Furosemide (Lasix) C. Check catheter for kinks D. Increase the oral intake to 30/hr

199. Kathy, who has partial-thickness burns on the face, inquiries about skin care after

discharge. Which of the following should you include in your discharge teaching?

A. Continue to eat high caloric high food for the next month B. Wear a pressure garment daily for one year C. Avoid sunlight for the next three months D. Avoid facial makeup for at least a year

200. You are caring for Lenard who sustained severe burn injury and he is in the emergent phase of burn injury. As his nurse, you gathered the following: Hemoglobin 13.5 g/100 ml, Hematocrit 50%, serum Na 130 mEq/L. How will you explain the laboratory results?

A. These are due to hemodilution from rapid IV fluid replacement B. All the laboratory tests are within normal rage C. They are slightly abnormal but will normalize once IV fluids have been started

Page 54: NLE Compilation 1

54

D. These are due to a loss of serum and interstitial fluid through the burn wound.

SUBMIT THIS TEST BOOKLET TOGETHER WITH THE ANSWER SHEET TO YOUR WATCHERS. BRINGING THE TEST BOOKLET OUT OF THE ROOM WILL BE A GROUND FOR CANCELLATION OF YOUR EXAMINATION.

***END*** NURSING PRACTICE V SET A ________________________________________________________________________ NURSING PRACTICE V – Foundation of PROFESSIONAL Nursing Practice GENERAL INSTRUCTIONS:

21. This test booklet contains 100 test questions. 22. Read INSTRUCTIONS TO EXAMINEES printed on your answer sheet. 23. Shade only one (1) box for each question on your answer sheets. Two or more boxes shaded

will invalidate your answer. 24. AVOID ERASURES. 25. This is PRC property. Unauthorized possession, reproduction, and/or sale of this test is

punishable by law. Per RA 8981. ________________________________________________________________________ INSTRUCTIONS:

13. Detach one (1) answer sheet from the bottom of your Examinee ID/Answer Sheet Set . 14. Write the subject title “Nursing Practice V” on the box provided. 15. Shade Set Box “A” on your answer sheet if your test booklet is Set A; Set Box “B” if your test

booklet is Set B. MULTIPLE CHOICE SITUATIONAL Situation 1 – The following questions refer to nurse’s efforts to do collaboration and teamwork. Select the best answer.

1. The most important role of the nurse as a member of the team is to:

A. carry out medical orders B. meet the needs for the physical well being of patients C. coordinate the psychological care and management of clients D. keep a 24 hour watch for the patients

2. A biological/medical approach to patient care utilizes which of the following?

A. Million therapy C. Behavioral therapy B. Somatic therapy D. Psychotherapy

3. Which of these nursing actions belong to the secondary level of preventive intervention?

Page 55: NLE Compilation 1

55

A. Providing mental health consultation to health care providers B. Providing emergency psychiatric services C. Being politically active in relation to mental health issues D. Providing mental health education to members of the community

4. When the nurse identifies a client who has attempt to commit suicide the nurse should:

A. call a priest B. counsel the client C. refer the client to the psychiatrist D. refer the matter to the police

5. The community health nurse was invited by the principal of an elementary school and was asked to give a talk to parents. An appropriate topic would be:

A. the legal aspects of drug abuse B. disciplining children at home and school C. marital crises D. the problems of out of school youth

Situation 2 – The nurse visited the Reyes family to check on their two growing children, aged 7 and 4 years. Upon her visit she observed that common areas of arguments between Mr. and Mrs. Reyes are about conflicting ways of bringing up their children. Mrs. Reyes is lax and tolerant while Mr. Reyes often insists strict ways to a point of protectiveness from what he perceives as unsafe i.e. community and neighbors that cannot be trusted.

6. Mr. Reyes remarked “I am wary about people visiting- with all the media news about child

kidnapping and robberies. “The nurse’s BEST response would be:

A. “Would you rather wish that I don’t come and visit you may regard me as a stranger?”

B. “I get that.” The nurse diverts the attention to talk about non-threatening topics. C. “It must be distressing to think and feel the way you do.” D. “I acknowledge what you are saying. My concern is the health care of your family

and information are strictly confidential.”

7. Mrs. Reyes expressed that her socializing with neighbors is limited because her husband thinks she is getting overly friendly with a guy next door. Which of the following would the nurse emphasize as basic?

A. Keeping trust in the relationship B. Avoid relating with neighbors to minimize conflict C. Be assertive to express to express her individuality D. Ignore the husband and just be supportive

8. For the nurse to be effective in developing rapport with the family it is essential that she keeps her appointment on time and stick to a care plan. She is applying the principle of:

A. responsibility and accountability B. consistency and predictability C. honesty and integrity D. empathy and compassion

9. Which of these symptoms if demonstrated by Mr. Reyes would necessitate referral to a doctor?

A. Hypervigilance C. hypersensitive B. Suspicious affect D. loss of reality contact

10. The paranoid client utilizes which of the following defense mechanisms?

A. Sublimation C. Rationalization B. Projection D. Reaction formation

Page 56: NLE Compilation 1

56

Situation 3 – Mr. Sison has been diagnosed as having early chronic glaucoma. He has been admitted to the hospital for treatment.

11. The nurse identified a nursing problem of disturbed sensory perception: visual impairment characterized by:

A. sudden loss of eyesight B. loss of night vision C. loss of peripheral vision D. loss of central vision

12. In order to understand the rationale for drug therapy, it is important for the nurse to know that glaucoma is usually caused by:

A. opacity in the lens B. gradual diminution of the retina C. damage to the proteins in the lens D. increase production of aqueous fluid

13. Diamox is a drug used in the treatment of glaucoma. Which of these is the effect of this drug?

A. Constricts the pupil B. Acts as osmotic diuretic C. Reduces the production of aqueous humor D. Facilitates outflow of aqueous humor

14. Public health nurses should identify which of these patients as a risk group for development of glaucoma, hence the need for annual eye examinations:

A. Patient with Parkinson’s disease B. Cancer patients C. Diabetic and hypertensive patients D. Patient with COPD

15. The appropriate method of instilling eye drops is: Instilling into an opened eye, with the head held back and with the eye looking:

A. Upward C. aggressiveness B. downward D. suspiciousness

Situation 4 – SEXUAL DISORDER

16. A hospitalized male adolescent flirts with and is sexually provocative toward a female nurse. The nurse can respond MOST therapeutically by doing which of the following?

A. Telling him she is married and too old for him B. Introducing him to female clients his own age C. Encouraging him to watch TV in his room D. Ignoring his flirtatious and provocative behaviors

17. The premorbid personality of a person with a non-psychotic maladaptive response to anxiety may most accurately be described as:

A. unpredictable, impulsive and aggressive B. rigid, insecure and conforming C. dependent, pessimistic and moody D. anxious, insensitive and self-absorbed

18. An oral-dependent personality is characterized by which of the following?

A. Helplessness C. Aggressiveness B. Hopelessness D. Suspiciousness

Page 57: NLE Compilation 1

57

19. The pedophile’s choice of a sex object is primarily based on:

A. difficulty relating with adults B. feelings of tenderness toward children C. fears of incestuous impulses D. preferred for a passive sexual role

20. A young adult male unable to stay put in one job and has no commitment in his relationship is having difficulty achieving a sense of:

A. Autonomy C. industry B. Trust D. intimacy

Situation 5 – Anita is experiencing rape-trauma syndrome in an acute phase. She had been invited to a fraternity party. She had too much drink and she has feelings of anger, humiliation, helplessness, nausea, vomiting, nightmare and muscle tension.

21. When the nurse approached Anita, initially she was just crying, felt she was in a nightmare

and she was at a loss. The appropriate nursing diagnosis is

A. Situational low self-esteem B. Sexual violence C. Ineffective coping D. Sexual dysfunction

22. Anita expressed to the nurse that she douched, showered for an hour and still did not feel clean. Anita is experiencing:

A. guilt C. denial B. anger D. frustration

23. Which of these communicate unconditional acceptance of Anita and hr situation?

A. “You are here and I am ready to listen.” B. Why did you date a guy you hardly knew?” C. Tell me when you are ready and I’ll come back to you.” D. I would be best of help if you stop crying.”

24. Anita is experiencing:

A. maturational crisis C. Social crisis B. developmental crisis D. frustration

25. Which of these behaviors of Anita signal her readiness to proceed to the working phase of the nurse-patient relationship?

A. She states she trusted the nurse B. She wants to talk to a lawyer C. She inquires about personal information about the nurse D. She wants to be told what her rights are

Situation 6 – The psychiatric mental health nurse adheres to standards that ensure quality improvement. The following situations and behaviors are means to achieve this goal.

26. This is a process wherein the client’s chart is reviewed to compare criteria for quality care

with actual practice:

A. Psychiatric Audit B. Nursing Care Process C. Interaction Process Analysis D. Algorithms

Page 58: NLE Compilation 1

58

27. In order to assess “Reliability” as a behavioral characteristic, the nurse would ask herself which of the following questions regarding her recording:

A. Did the history of the present problem correlate with the review of growth and

development? B. How long did it take to complete the nursing data base? C. Is the nursing data base complete? D. Are the nursing history and psychosocial assessment accurate?

28. All of these are the advantages of peer review EXCEPT:

A. Demands accountability for nursing actions B. Has the possibility of enhancing intra professional respect C. It requires the development of standards for quality care D. Provide an evaluation of the nurse’s abilities

29. The nursing team leader wants to involve all the nurses in participating in their own personal and professional growth through a brainstorming session. One of the most important ground rules is:

A. Follow the problem solving approach B. Do not pass judgment on the ideas presented C. Ideas must be feasible D. Suggestions must be cost effective

30. “Did the nurse perform in the best possible manner without waste?” aims to describe the

nurse’s:

A. thoroughness C. efficiency B. reliability D. analytic sense

Situation 7 – A nurse was interested to study the research question: “What are the

differences and similarities between aggressive and non-aggressive cognitively impaired, elderly, institutionalized people?”

31. Investigation of cognitively impaired individual presented some ethical dilemmas. Which of

the following protocol would be considered unethical?

A. Recording interaction with the elderly with their permission B. Verbal permission from the subject is unnecessary C. Data coded and recorded solely by the investigation D. A written consent from the institution and a significant other

32. A semi-structured interview was conducted. This means that:

A. Interview is conducted precisely in the same manner B. Interviewer is not held to any specific question C. Subject is allowed to express without any suggestion from interviewer D. Interviewer is free to probe beyond a number of specific major questions

33. The type of study conducted is:

A. Descriptive C. experimental B. Quasi-experimental D. case study

34. The review of literature included reference to retrospective studies. Such studies have the advantages EXCEPT:

A. Data are inexpensive to obtain B. Possibility of memory bias and distortion of fact C. There is much material available D. It is easy to get data

Page 59: NLE Compilation 1

59

35. The average age of the respondents was 86. this represents:

A. the sum ages divided by total number of participants B. the youngest participant is 86 years old C. the oldest participant is 86 years old D. most of the number participant is 86 years old

Situation 8 – Mr. David is brought to the hospital due to pain radiating to the hip and leg. He is diagnosed with a herniated lumbar disk. H is scheduled for myelogram.

36. After the procedure, the nurse must include which of th following nursing action in his

care? A. Assess for movement and sensation of the lower extremity B. Place the client in most comfortable position C. Lying supine with heels flexed D. Bed rest with bed elevated at 45 degrees

37. Mr. David is scheduled for lumbar laminectomy. Post operatively the nurse should:

A. Logroll the client with the help of another nurse B. Inform the client that he should be in supine position C. Assess the sensory loss in his legs D. Instruct the patient to move from side to side

38. Trimethobenzamine Hydrochloride (Tigan) was administered postoperatively. The action of this drug is effective when it:

A. Controls nausea C. controls muscle spasm B. Controls pain D. controls edema

39. Mr. David is to ambulate for the first time following surgery. What nursing action should be BEST when the client begins to faint?

A. Get another nurse for help B. Maneuver the client to a sitting position C. Get back to his bed and place in side lying position D. Assist the client to form a wide base of support and lean against the nurse

40. Mr. David has to wear back brace. Which position is recommended when the brace is applied?

A. Sitting position C. lying on his side in bed B. Standing position D. supine position in bed

Situation 9 – Through the nurse-patient relationship, the nurse intervenes utilizing effective communication techniques. The following are varied situations in a psychiatry ward.

41. The patient verbalizes, “Masama and pakiramdam ko. Hindi ako nakatulog kagabi.“A

therapeutic response of the nurse would be:

A. “Baka ini-istorbo ka na naman ng mga boses.” B. „Sinabi mo sana sa nars nabigyan ka ng sedative mo.“ C. Relax lang! Huwag ka masyadong mag-iisip ng mga problema mo.“ D. “Maari mo bang sabihin sa akin and mga naiisip at nararamdaman mo?“

42. Soledad is terminally ill of cancer. Looking sad, she expresses, “Wala na yata akong pag-asang mabuhay pa,” A response which fosters hope is:

A. “Mukhang napakabigat ang dinaramdam ninyo. Andito po ako at puwede

tayong mag-usap.“

Page 60: NLE Compilation 1

60

B. “Huwag po ninyong isipin ang sakit ninyo. Bale wala yon. Andito naman ako para makausap ninyo.”

C. “Lakasan ang loob ninyo. Lahat naman po tayo ay doon ang patutunguhan.” D. “Gagaling din po kayo. Huwag po kayong mag-aalala.”

43. Camilia verbalizes, “Pinag-uusapan nila ako. Ayaw nila ako.“ A therapeutic response is:

A. “Nalulungkot ba ang pakiramdam mo?” B. “Hayaan mo sila. Ang mahalaga ay ang palagay mo sa sarili mo.” C. “Sino ang ‘nila’ na tinutukoy mo?” D. “Huwag mong isipin yan. Hindi tama yan.”

44. During socialization, Nicanor was provoked, became furious and started shouting “Walang hiya kayo! Ako ang bida dito!” The nurse’s action is:

A. Take him away form the group until he manages to have control of himself. B. Immediately restrain him and put him on isolation to protect other patients. C. Prevent him from becoming more furious by giving an extra PRN dose of sedative. D. Respond with, “Nicanor, pare-pareho lang kayo ng mga ibang pasyente dito.”

45. Nicanor becomes verbally assaultive to the nurse. He says, “Ikaw, nurse, wala kanga lam! Marunong pa ako sa iyo e. Ano ba ang ipinagmamalaki mo!” The nurse responds therapeutically by:

A. admonishing him with, “Ako ang nurse dito. Dapat sumunod ka sa akin.” B. Acknowledging his behavior, however, put him in his right senses ; respond with,

“Oo nga, galit ka sa nurse pero hindi tama na naninigaw ka.” C. Acknowledging his behavior and respond, “Nagagalit ka sa nurse at

nawawala ka ng control sa sarili mo.” D. Ignoring the behavior of the patient

Situation 10 – Nicanor was discharged form the hospital and recovered from a manic episode of Bipolar Disorder. NIcanor was readmitted with an entirely different behavior. he was very depressed.

46. The defense mechanism utilized by manic patients to cover up depression is:

A. reaction formation C. displacement B. compensation D. denial

47. The psychodymanics of depression is:

A. lax super-ego B. weak super-ego C. internalized hostility feelings D. narcissistic personality

48. Which of these drugs is likely to indicated to Nicanor?

A. Serenace (Haloperidol) B. Valium (Diazepam) C. Tofranil (Imipramine HCl) D. Trilaton (Pherpenazine)

49. Therapeutic use of self is essential in relating with psychiatric patients. This is BEST demonstrated in:

A. sympathizing with the miserable feelings of Nicanor B. engaging Nicanor in productive activity C. engaging NIcanor in introspective thinking D. suppressing her own feelings toward NIcanor

50. After three days of antidepressant medication, Nicanor still manifests depression. The nurse evaluates this as:

Page 61: NLE Compilation 1

61

A. unusual because action of antidepressant drug is immediate B. expected because it takes about two weeks for the medication to be effective C. unexpected because it takes within one week for the medication to be effective D. ineffective because perhaps the drug’s dose is inadequate

Situation 11 - Ninety year old Purita is confined at the medical unit for respiratory ailment for which a breathing apparatus is prescribed for her to use while she sleeps. She refuses to wear it continuously though she fully understands the medical indication for it.

51. Which of these etical principles can guide the nurse in her action?

A. Beneficence C. Autonomy B. Fidelity D. Nonmaleficence

52. Purita has six children who are already adults. They differ in their opinion whether or not to allow their mother to decide for herself. The nurse would encourage family conference for:

A. the eldest child’s opinion to be given priority B. majority of the children to decide C. allowing the medical staff to decide in their behalf D. consensus building

53. Breathing treatments are to be given to Purita. In anticipation that Purita might refuse,

Dinio, one of the children requests that he be the one to sign the consent in behalf of their mother. The nurse explains that Purita is rational in her thinking and which of these client’s right must be regarded?

A. Right to refuse treatment B. Right to privacy C. Right to informed consent D. Right of habeas consent

54. Which of these would be the nurse’s priority following the treatment principle of least

restrictive alternative?

A. One of one staffing C. Physical restraint B. Use of on site guard/watcher D. Seclusion

55. Purita talks about her joy in having responsible and accomplished children and recalls challenging career as a lawyer. She is demonstrating a sense of:

A. ego integrity C. generativity B. industry D. autonomy

Situation 12 – Marina, 26 years old, is aloof in relating with other patients and members of the staff. She claims that the medications being given her are meant to poison her. She is also suspicious about the food being served for her.

56. Basically, Marina is suspicious because of her inability to develop a sense of:

A. Intimacy C. Trust B. Generativity D. Intiative

57. Marina utilizes projection by being suspicious. This means that she:

A. unconsciously refuses to accept a feeling, thought or impulse and attributes it to someone else

B. justifies behavior, attitudes and feelings with excuses C. involuntarily refuses to acknowledge reality D. involuntarily excludes wishes, impulses, memories and feelings from awareness

Page 62: NLE Compilation 1

62

58. Which of these nursing approaches is MOST appropriate for the nurse to begin with?

A. Engage Marina for at least one hour in a one-to-one interaction daily B. Invite her to socialize with other patients C. Make self available while maintaining distance until patient shows readiness

to interact D. Refer her for activity therapy

59. When she resists to take her medication, it is best to:

A. let her read the drug literature to convince her that it is therapeutic B. force her to take the drug to maintain therapeutic effectiveness of the drug C. have the same nurse, who she interacts with regularly, administer the drug D. request the doctor to give her medication

60. Another reason why she refuses to take Thorazine is because she complains of robot like movement and slurred speech. The nurse’s action is:

A. decrease the dosage of thorazine B. explain the extrapyramidal side effects and administer Benadryl C. avoid giving foods that are rich in tyramine D. withhold medication until referral is made to the doctor

Situation 13 – The supervising nurse received report that a staff nurse is displaying frequent irritation, anger, and even indifference toward clients and co-workers.

61. The initial action of the supervisor would be to:

A. post guidelines on proper decorum of nurses in the bulletin board B. write a memo of warning to the nurse C. request anecdotal report form nurse’s co-workers D. call the nurse for a one on one conference

62. The nurse expressed increasing feelings of dissatisfaction. The supervising nurse intervenes therapeutically by taking the role of:

A. administrator by relieving her of responsibilities B. therapist by delving into the nurse’s internal conflicts C. counselor by actively listening D. educator by reorienting her of her role as a nurse

63. Coupled with poor work performance, mental and physical fatigue and actual withdrawal from client contact and nursing duties, the nurse can be said to be suffering from:

A. psychotic anxiety B. staff burnout C. personality maladjustment D. neurotic depression

64. A priority in the nurse’s personal development would be to:

A. address her physical well-being B. boost her self-confidence C. provide social support D. help her find value and meaning in her work

65. The most relevant professional program for her would be:

A. assertiveness training B. stress management C. group dynamics and team building D. behavior modification

Page 63: NLE Compilation 1

63

Situation 14 – The purpose of the nursing care plan is to identify the care for an individual patient based on his problems. The nurse writes a nursing care plan for a patient based on nursing care standards.

66. Given this example of a problem, “Anxiety due to a job interview”. The “due to” or the

reason for the problem should be included if it is known. The initial step in identifying problems is:

A. gather data about the patient B. determine if the problems are usual or unusual C. analyze the data D. analyze the problems as concisely as possible

67. Given this example of an expected outcome: “Openly verbalize anxiety about job interview. Identify how he can prepare for the job interview.” Which of these is not a criterion of expected outcomes?

A. An expected outcome is stated in terms of what the patient will do B. An expected outcome is stated in terms of what the nurse will do C. Every outcome must be measurable D. Every outcome answers the question “How will you know when the problem is

resolved?”

68. The following are reasons for setting deadlines within which to achieve outcomes of care EXCEPT:

A. Indicate specific times to review progress or lack of progress B. Does not allow plans to be changed C. Allow plans the need to be changed D. Set the time by which the expected outcome should be reached

69. Which of these is not a relevant nursing order?

A. Ask the patient any untoward side effects of medications he is taking B. Have patient role play interview situation C. Discuss with a patient with specific means he might prepare for the job interview D. Ask the patient what he is feeling about the job interview

70. Which of these practices on evaluation support nursing care? Review of care plan is:

A. a nursing team responsibility B. the sole responsibility of the primary nurse C. the responsibility of peers D. the sole responsibility of the supervisor

Situation 15 – A nurse assigned in the neurologic unit is taking care of clients with varying degrees of degenerative disorders.

71. Ma. A with myasthenia gravis is having difficulty speaking. What communication strategies

should the nurse avoid when interacting with Mr. A?

A. Repeating what the client says for better understanding B. Using paper and pencil in communicating with the client C. Encouraging the client to speak slowly D. Encouraging the client to speak quickly

72. When planning for nursing care for Mr. B who has Parkinson’s disease, which of the following goals would be MOST appropriate?

A. to improve muscle tone B. to start rehabilitation as much as possible C. to treat the disease

Page 64: NLE Compilation 1

64

D. to maintain optimal body function

73. For the past 10 years, Alma, 42 years old, has had multiple sclerosis. Clients with multiple sclerosis experience many different symptoms. As part of the rehabilitation planned for Alma, the nurse suggested therapy and hobbies to help her:

A. strengthen muscle coordination B. establish routine C. develop perseverance and motivation D. establish good health habits

74. On his second day of hospitalization, Mr. Santos was unable to stand and is having difficulty swallowing and talking. Which of the following is the priority of the nurse in assisting Mr. Santos?

A. To prevent bladder distention B. To prevent decubitus ulcer C. To prevent contracture D. To prevent aspiration pneumonia

75. The wife of a seventy two (72) year old male with a diagnosis of Alzheimer’s disease begins to cry and tells the nurse, “I could not understand my husband anymore. He has changed drastically.” Which of the following responses of the nurse is MOST appropriate?

A. “The physician and the staff will make sure that your husband will be comfortable

and safe here.” B. “This has been a difficult time for you. Let us walk and find a quiet place

where we can talk.” C. “He will soon recover in his condition.” D. “You need not worry, we are doing the best we could.”

Situation 16 – Annie has a morbid fear of heights. She asks the nurse what desensitization therapy is:

76. The accurate information of the nurse of the goal of desensitization is:

A. to help the clients relax and progressively work up a list of anxiety provoking situations through imagery.

B. To provide corrective emotional experiences through a one-to-one intensive relationship.

C. To help clients in a group therapy setting to take on specific roles and reenact in front of an audience, situations in which interpersonal conflict is involved.

D. To help clients cope with their problems by learning behaviors that are more functional and be better equipped to face reality and make decisions.

77. It is essential in desensitization for the patient to:

A. have rapport with the therapist B. use deep breathing or another relaxation technique C. assess one’s self for the need of an anxiolytic drug D. work through unresolved unconscious conflicts

78. In this level of anxiety, cognitive capacity diminishes. Focus becomes limited and client experiences tunnel vision. Physical signs of anxiety become more pronounced.

A. severe anxiety C. mild anxiety B. panic D. moderate anxiety

79. Antianxiety medications should be used with extreme caution be cause long term use can lead to:

A. Parkinsonian like syndrome

Page 65: NLE Compilation 1

65

B. Hypertensive crisis C. Hepatic failure D. Risk of addiction

80. The nursing management of anxiety related with post traumatic stress disorder includes all of the following EXCEPT:

A. encourage participation in recreation or sports activities B. reassure client’s safety while touching client C. speak in a calm soothing voice D. remain with the client while fear level is high

Situation 17 – For personal and professional development, the nursing staff decided to hold a staff development program, “Self-enhancement through Assertiveness”.

81. An appropriate assessment tool to maximize gathering of needs of nurses is through:

A. interview of nurses C. observation B. survey D. brainstorming session

82. A priority objective of the program is:

A. develop the art of public speaking B. project a positive image of the nursing profession C. develop art and skills of therapeutic use of self D. earn continuing education units

83. The most effective way to practice assertiveness skills is through:

A. written evaluation form C. descriptive report B. process recording D. role play

84. The least satisfactory method to evaluate the effectiveness of the program is through:

A. group discussion and report B. return demonstration C. attendance D. individual interviews

85. Which of these feedback from individual participants indicate maximum gain from the staff development program?

A. “I will write a plan for personal development program.” B. “I feel very good. The program inspired me a lot.” C. “I learned a lot. I hope to have more seminars of its kind.” D. “I have a “Do it Now” project for myself i.e. to approach my clinical

supervisor regularly to discuss nursing care of our clients.” Situation 18 – A vehicle hit some pedestrians while waiting for a bus ride. Some of the victims suffered injuries in the different part of their bodies. The victims were brought to the nearby hospital. One of the victims, Josephine was confirmed to have a fractured left arm. While waiting for the plaster cast to be applied, Josephine appears to be anxious.

86. To reduce anxiety, the nurse teaches the procedure to the client. which of the following

topics should NOT be included in the teaching plan?

A. Leave the cast uncovered to promote drying. B. Bear weight on the plaster cast for one hour. A stockinet will be placed over

the left arm to be placed in cast. C. Handle hardening cast with palm of hands D. Trim and reshape finish cast with knife or cutter.

Page 66: NLE Compilation 1

66

87. Cast was applied on Josephine’s left arm. In assessing the neurovascular status of the client, which of the following assessment findings should be reported to the physician?

A. Pain on the left arm B. Swelling of the fingers C. Skin abrasions on the edges of the plaster cast D. Nail bed capillary refill time of 10 seconds

88. One of the victims, a sixty year old woman sustained hip fracture. Prior to surgery, a Buck’s extension traction is to be applied. The rationale of traction is primarily based on the understanding that Buck’s extension traction:

A. reduces muscle spasms and helps to immobilize the fracture B. allows reduction of the fracture site for bone healing. C. Secures the fracture site to prevent damage to the muscle tissues D. Secures the fracture site for rigid immobilization

89. Phillip was placed in skeletal leg traction with an overbed frame. He is not allowed move from side to side. Which of the following nursing interventions is useful in maintaining effective traction?

A. Assist the client by holding the trapeze and raising the hips off the bed. B. Check the apparatus, that weights hang free and knots in the rope are tied

securely C. Suspend the trapeze within easy reach of the client D. Support the affected extremity while the weights are removed.

90. To prevent complications when a child is in Buck’s traction, the nurse should”

A. clean the extremity and keep the skin dry B. assess any skin and circulatory disturbances C. clean the pin sites as necessary D. provide high fiber small meals

91. All of the following concepts are true EXCEPT:

A. Hostility is destructive B. Frustration develops in response to unmet needs, wants and desire C. Anger is incompatible with love D. Aggression can be expressed in a constructive as well as a destructive manner.

92. Carlo is acting out hostile and aggressive feeling by kicking the chairs in the room. the MOST effective way to deal with Carlo’s behavior is initially to:

A. set limits on the behavior by verbal command B. administer PRN tranquilizer C. remove the chairs from the room D. restrain the patient and place him in the “Isolation Room”

93. Mrs. Dizon was visiting her son at the Psychiatry Ward. Which of the following items will the nurse not allow to be brought inside the ward?

A. string rosary bracelet C. bottle of coke B. box of cake D. rubber shoes

94. Which of the following will probably be most therapeutic for a patient on a behavioral modification ward?

A. if the client is agitated, discuss the feelings especially anger B. insist to stop obscene language by verbal reprimand C. give client support and positive feedback for controlling use of obscene

language D. Provide a punching bag as an alternative to express upset emotions

Page 67: NLE Compilation 1

67

95. Which of the following must be considered while planning activities for the depressed patient?

A. activities which require exertion of energy B. challenging activities to get him out of his depression C. variety of structured activities D. variety of unstructured activities

Situation 20 - Jim, age 25, recalled that his problem began around age 15-16. He would count pencils in a mug over and over with the thought that stopping could result in something bad happening.

96. There are many things Jim seems he has to do to keep himself from feeling:

A. confused C. excited B. suspicious D. anxious

97. He has change clothes 20 times before work, chew each bite he east 24 times and go up and down the stairs four to five times before it feels right. He is demonstrating:

A. ideas of reference B. denial and projection C. obsession and compulsion D. rationalization and over reaction

98. The objective of nursing care for Jim is to develop or increase feelings of:

A. self-mastery C. self-actualization B. self worth D. self-determination

99. All of these are therapeutic interventions EXCEPT: A. impose limits every time the behavior becomes repetitive B. establish a routine for him C. assign task that can be done repetitively D. facilitate self-expression

100. Jim is aware of his behavior yet realizes that it is very disturbing to him. This is a pattern of:

A. personality disorder C. neurosis B. psychosis D. habitual disorder

SUBMIT THIS TEST BOOKLET TOGETHER WITH THE ANSWER SHEET TO YOUR WATCHERS. BRINGING THE TEST BOOKLET OUT OF THE ROOM WILL BE A GROUND FOR CANCELLATION OF YOUR EXAMINATION.

***END***